You are on page 1of 118

GYNECOLOGY

ANATOMY AND EMBRYOLOGY

Directions: Each of the numbered items or incomplete statements in this section is


followed by answers or by completions of the statement. Select the ONE lettered answer
or completion that is BEST in each case.

1- The following about the round ligament of the uterus is (are) correct EXCEPT:
a)
b) Passes into the inguinal canal.
c) d) Is derived from the gubernaculum.
e) Is continuous with the ovarian ligament at the cornu of the uterus.

2- The body of the uterus drains to the following nodes EXCEPT:


a) Sacral.
b) Superficial inguinal.
c) Obturator.
d) Femoral.
e) External iliac.

3- The lymphatic drainage of the cervix is to the following lymph nodes EXCEPT:
a) The femoral lymph nodes.
b) The internal iliac lymph nodes.
c) The para-cervical lymph nodes.
d) The pre-sacral lymph nodes.
e) The Obturator lymph nodes.

4- The following is correct about the ovarian ligaments:


a) Contain ureters.
b) Contain ovarian arteries.
c) Are attached laterally to pelvic wall.
d) Lie anterior to the broad ligament.
e) Are homologous to part of the gubernaculums testis in the male.
5-
6- AH the following are correct about the pelvic ligaments and fascia EXCEPT:
a) The cardinal ligaments arise from the inferior pubic rami.
b) Peritoneum covers the superior portion of the post vaginal wall.
c) The suspensory ligament of the ovary is superior to the round ligament.
d) The round ligament contains lymphatic vessels.
e) The ureter penetrates the cardinal ligament.

7- In the development of the ovaries, all are correct EXCEPT:


a) The structures in the ovaries have different origins.
b) Involve migration of the germ cells.
c) Involve migration of the ovaries.
d) The homologous structure in the male is the testis.
e) Develop at the Gubernaculum.

8- The presence of a uterus in a phenotypically normal male is due to:


a) Lack of mullerian inhibiting factor.
b) Lack of testosterone.
c) Increased levels of estrogens.
d) 46, XX karyotype.
e) Presence of ovarian tissue early in embryonic development.

9- Pregnancy complications that can occur due to bicornuate uterus include the
following EXCEPT:
a) Abortion.
b) Malpresentation.
c) Operative delivery.
d) Twin pregnancy.
e) Premature labor.

10-Possible clinical consequences of congenital malformations of the female genital


system include all of the following EXCEPT:
a) Habitual abortion.
b) Congenital fetal malformations.
c) Ectopic pregnancy.
d) Oblique lie of the fetus.
e) Expulsion of intrauterine devices.
11-Imperforate hymen might present with the following EXCEPT:

:
a) Primary amenorrhea.
b) Cyclic menstrual molimina.
c) Acute retention of urine.
d) pelvi-abdominal mass.
e) Hypomenorrhea.
12-The clinical findings of imperforate hymen include the following EXCEPT:
a) Hirsutism.
b) Cryptomenorrhea.
c) Hematometra
d) Pelvi-abdominal swelling.
e) Retention of urine.
13-

14-Ambiguous genitalia at birth may be associated with the following EXCEPT:


a) congenital adrenal hyperplasia.
b) maternal ingestion of danazol during pregnancy.
c) complete androgen insensitivity.
d) true hermaphroditism.
e) karyotype of 46 XX.

15-Bicornuate uterus might predispose to:


a) Recurrent preterm labor.
b) Primary amenorrhea.
c) Recurrent oblique lie.
d) Retention of the placenta after delivery.
e) Menorrhagia.

16-The following is derived from gubernaculum in female:


a) Ovarian ligament.
b) Mesovarium and ovarian ligament.
c) Ovarian ligament and broad ligament.
d) Round ligament and broad ligament.
e) Mackenrodt's ligament and ovarian ligament

17-All are vestigial remnants EXCEPT:


a) Kobelt's tubules.
b) Epoophoron.
c) Tubal fimbria.
d) Paraoophrorn.
e) Gartner duct.
18-

19-Urogenital sinus gives the following:


a) Urinary bladder.
b) Skene's ducts.
c) Lower part of the vagina.
d) The ureteric bud.
e) Bartholin's glands

20-Barr body may be found in the following EXCEPT:


a) Klinefilter's syndrome.
b) Turner's syndrome.
c) Down's syndrome.
d) Adrenogenital syndrome.
e)

21-Congenital absence of the vagina is most commonly associated with:


a) Absent secondary sexual characteristics.
b) Absent uterus.
c) Exposure to diethyl stilbosterol in utero.
d) Turner's syndrome.
e) Imperforate anus.

Directions (Questions 22 through 46) ach set of matching questions in this section
consists of a list of 5 to 8 lettered options followed by several numbered items. For each
item, select the ONE best lettered option that is most closely associated with it. Each
lettered heading may be selected once, more than once, or not at all.

Questions 22-26
For each of the following epithelium, select the organ that is lining it:
a) Stratified squamous
b) Pseudo-stratified
c) Transitional
d) Columnar

22-Dome of the urinary bladder


23-External urethral meatus
24-Endocervix
25-Adult vagina
26-Endometrium

Questions 32-36 MCQ


Match the homologous corresponding structures in both sexes
a) Testis
b) Ventral aspect of the penis
c) Penis
d) Scrotum
e) Gubernaculum testis
f) Cremasteric muscle
g) Glans penis

32-Ovarian ligament
33-Labia majora
34-Labia minora
35-Clitoris
36-Ovary

Questions 37-41
Match the origin of the organ with the organ given
a) Mesonephric duct
b) Genital tubercle
c) Gubernaculum
d) Urogenital fold
e) Paramesonephric duct
f) Urogenital sinus

37-Fallopian tube
38-Ovarian ligament
39-Labium minus
40-Clitoris
41-Uterus
Questions 42-46

Directions: Each of the following numbered items or incomplete statements in this


section is followed by numbered items that in certain combination leads to the BEST
lettered combined answer or/and completion. According to the following directions,
choose the BEST lettered answer:
If
1,2,3 are correct, choose A
1. 3 are correct, choose B
2. 4 are correct, choose C
1, 2, 3, 4 are correct, choose D
All are correct, choose E

47-In the female genitalia:


1. The anterior and posterior walls of the empty vagina lie in apposition.
2. The urethra opens in the vestibule.
3. Bartholin's gland lies deep to the vestibular bulb.
4. Secretions arise from vaginal epithelial glands.
5. The labia minora have hair follicles.

48-The nerve supply of the vulva and perineum is derived from the following nerves:
1. Pudendal.
2. Genito-femoral.
3. Posterior cutaneous of thigh.
4. Obturator.
5. Anterior cutaneous of thigh.
49-The vulval blood supply is via the:
1. Internal pudendal artery.
2. Vaginal artery.
3. External pudendal artery.
4. Superior vesical artery.
5. Middle rectal artery.

50-The major supports of uterus are:


1. The transverse cervical (cardinal) ligaments.
2. The uterosacral ligaments.
3. The pubo-cervical ligaments.
4. The infundibulo-pelvic ligaments.
5. The round ligaments.

51-The lymphatic drainage from the uterine cervix is through the following lymph
nodes:
1. External iliac.
2. Obturator.
3. Pre-sacral.
4. Superficial femoral.
5. Superficial inguinal.
52-
53-
54-
Directions: In the following questions, each item has 5 lettered options each of which
could be a true or a false statement. Tick each of these options according to whether
they are false or true

55-The vagina:
a) Contains mucus secreting glands.
b) Relates posteriorly to the rectum in its middle third.
c) Relates anteriorly to the bladder base in its lowest third.
d) Is supplied in part by the uterine artery.
e) Is entirely derived from the MULLERIAN duct.

56-The ovary:
a) Is derived from THE GENITAL TUBERCLE
b) Is attached to the uterine cornu by the infundibulo-pelvic ligament.
c) Lies posteriorly to the board ligament.
d)
e) Is attached to the distal portion of the fallopian tube.

57-
a)

58-The cervix:
a)
b)
c) Has columnar epithelium lining the canal.
d) Produces a thick scanty discharge at ovulation.
e) Has the same proportion of muscle in its wall as the corpus uteri.

59-Imperforate hymen:
a) May lead to hematocolpos.
b)
c) Is frequently associated with renal abnormalities.
d)
e)

60-The Following statements about the lower part of the uterus are correct:
a)
b) The uterine artery passes inferior to the ureter.
c) A parous external os is slit-shaped.
d) Peritoneum passes to cover the upper part of vagina posteriorly.
e)
61-In the female pelvis, the peritoneum covers:
a) The whole of the anterior wall of the uterus.
b) The whole of the posterior wall of the uterus.
c) The fallopian tube partially.
d) The ovary fully.
e) The upper half of the posterior wall of the vagina.

62-The Fallopian Tube:


a) Possesses a ciliated lining.
b) Is actively motile.
c) Is easily palpable on bimanual examination.
d) Is covered totally by peritoneum.
e) Has a thick muscle layer in ampulla.

63-The ovary:
a) Lies posterior to the broad ligament.
b) Is covered by peritoneum.
c) Receives blood supply from a branch from the internal iliac artery.
d) Right ovarian vein drains directly into inferior vena cava.
e) Has lymphatic drainage to the para-aortic lymph nodes.

64-The uterus:
a) The uterine index in adult uterus is 1.
b) Anteflexion is maintained mainly by the tone of the uterosacral ligament.
c) The round ligament is attached to the anterior surface just below the cornu.
d) The uterine artery passes below the ureter (in the ureteric canal) to reach the uterus.
e) The uterus receives blood supply from the ovarian artery.
REPRODUCTIVE ENDOCRINOLOGY, INFERTILITY AND
MENSTRUAL DISORDERS

Directions: Each of the numbered items or incomplete statements in this section is


followed by answers or by completions of the statement. Select the ONE lettered answer
or completion that is BEST in each case.

65-The functions of mid-cycle LH surge include the following EXCEPT:


a) Enhances thecal cell androgen production.
b) Luteinizes granulosa cells.
c) Initiates resumption of meiosis.
d) Facilitates oocyte expulsion.
e) Produces luteolysis.

66-Follicle-stimulating hormone is elaborated by:


a) Chromophobe cells of the adenohypophysis.
b) Basophilic cells of the adenohypophysis.
c) Acidophilic cells of the adenohypophysis.
d) Follicular granulosa.
e

67-The following is a normal finding on the twenty-third day of a normal twenty-eight


day menstrual cycle provide strong evidence that ovulation has occurred:
a) Cervical mucus with positive arborization if dried on a glass slide.
b) Subnuclear vacuolation apparent on endometrium biopsy.
c) High cornification index in vaginal cytology preparations.
d) High level of total estrogens in a twenty-four hour specimen of urine.
e) Presence of an apparently healthy corpus luteum in an ovary.

68-The average blood loss resulting from menstruation is:


a) 10 to 15ml.
b) 25 to 50 ml.
c) 75 to 100ml.
d) 101 to 125ml.
e) 130 to 150ml.

69-All the following about precocious puberty is true EXCEPT;


a) It may involve menarche before the age of 8 years.
b)
c) Constitutional factor is rare.
d) It may be due to serious tumors.
e) It may be associated with contra-sexual development.

70-All the following about precocious puberty is true EXCEPT:


a) It is usually constitutional.
b) It may be associated with long bone fractures.
c) Pubic hair is first sign.
d) It can be delayed by GnRH analogues.
e) It results in adult short stature.

71-Menopause could be diagnosed by:


a) History of absent menstruation.
b) High FSH in young age group.
c) Absent menstruation for 1 year at the age of 52 year.
d) Hot flushes & lack of sleep.
e) Flushes lack.

72-Premature menopause is associated with the following EXCEPT:


a)
c) Radio therapy.
d

73-Facts about clomiphene citrate include the following EXCEPT:


a) has antiestrogenic properties.
b) may be started with a 50-mg dose daily.
c) is useful in primary ovarian failure.
d) may cause cervical mucus hostility.
e) may result in multiple pregnancy is a side effect of its use.

74-Complications that could be attributed to post-menopausal hormonal changes


include the following EXCEPT:
a) Procidentia.
b) Coronary heart diseases.
c) Cancer breast.
d) Stress urinary incontinence.
e) Depression.

76-In Hirsutism, the following is true EXCEPT:


a) It may be treated by dexamethasone when there is an adrenal cause.
b) Cyproterone acetate results in reduction of libido.
c) d) 99% of circulating testosterone is protein bound.
e) Raised

77-In severe ovarian hyperstimulation syndrome (OHSS):


a) Pleural effusion may occur.
b) Intravascular clotting can be a complication.
c) Hypoproteinemia may develop.
d) Bilateral oophorectomy is indicated.
e) Paracentesis may be of benefit.

78-Hyperprolactinemia may be associated with the following EXCEPT:


a) Chronic renal failure.
b) Galactorrhea.
c) Cimetidine therapy.
d) Methyl dopa therapy.
e) Adrenogenital syndrome.

79-Galactorrhea (non-gestational lactation) may result from the following EXCEPT:


a) Pituitary adenoma.
b) Hypothyroidism.
c) Renal failure.
d) Intrapartum hemorrhage.
e) Bronchogenic carcinoma.

80-Thc following is true about gonadotrophin releasing hormone (GnRH) analogues:


a) They are given continuously in vitro fertilization cycles (IVF) to stimulate follicular
growth.
b) They may cause endometrial hyperplasia.
c) They can be given intramuscular.
d) Their long term use increases bone density.
e) Progesterone back-up is better used with its chronic use.

81-The following is true about danazol EXCEPT:


a) It is a synthetic compound.
b) It has an androgenic properties.
c) It has an anabolic effects.
d) It has an antigonadotrophic effects.
e) The main indication is in the treatment of dysfunctional uterine bleeding.

82-The following is true about female sexual dysfunction EXCEPT:


a) Females have no refractory phase (unlike males).
b) Dyspareunia is mainly introital.
c) Fear of pregnancy and sexual transmitted diseases can be a cause of dyspareunia.
d) Urine loss at orgasm suggests stress incontinence.
e) Pregnancy may cause situational anorgasmia.

83-In vitro fertilization (1VF):


a) Down regulation of the pituitary gland is achieved by the use of human menopausal
gonadotrophins (HMG).
b) IVF is indicated for treatment of luteal phase defect in infertility.
c) IVF is indicated for treatment of unexplained infertility.
d) Oocyte retrieval is performed 28 hours after human chorionic gonadotropin (hCG)
injection.
e) Success rates are increased with increasing age.

84-Accepted parameters for a normal semen analysis are:


a) A sperm count of 10-20 million per ml.
b) A volume of 1 ml.
c) 60% motility.
d) 60% abnormal forms.
e) A positive mixed agglutination reaction.
85-The general incidence of infertility is:
a) 5%.
b) 10-15%.
c) 25%.
d) 30%.
e) 35%

86-According to WHO (world health organization), the percentage of normal


morphology of normal semen analysis is:
a) > 30%.
b) < 30%.
c) >50%.
d) > 70%.
e) > 90%

87-The following are advantages of laparoscopy in the diagnosis of infertility EXCEPT:


a) Good evaluation of tubal factor.
b) Diagnosis of endometriosis.
c) Diagnosis of Asherman's syndrome.
d) Good visualization of pelvic adhesions.
e) Direct visualization of corpus luteum.

88-The following statements are true as regards serum progesterone measurement in


diagnosis of infertility EXCEPT:
a) The normal level is 5 ng/ml.
b) Measurement is usually done at day 21 of the cycle.
c) Can be used for assessment of the luteal phase defect.
d) The normal level is variable between individuals.
e) Used in diagnosis of anovulation

89-The character of normal cervical mucus at the time of ovulation is:


a) Highly viscous and turbid.
b) Spinnbarkeit test is less than 6 cm.
c) Contains low amount of crystals of sodium and potassium chloride.
d) Positive ferning test.
e) Acellular

90-Polycystic ovarian disease characterized by all of the following EXCEPT:


a) Increase LH level.
b) Increase in androgen levels.
c) Decrease in estrogen levels.
d) Oligomenorrhea.
e) Android obesity

91-Hysteroscopy is important for diagnosis of the following factors in infertility


EXCEPT:
a) Intrauterine adhesions.
b) Submucous fibroids.
c) Peritubal adhesions.
d) Cornual block.
e) Septate uterus

92-Dysfunctional uterine bleeding (DUB) is frequently associated with:


a) Endometrial polyps.
b) Anovulation.
c) Cervicitis.
d) Systemic lupus erythematosus.
e) Von Willebrand's disease.

93-Treatment of dysfunctional uterine bleeding include the following EXCEPT:


a) Progestin therapy.
b) Estrogen therapy.
c) Danazol.
d) Endometrial ablation.
e) Bromocriptine
94-True statements concerning anorexia nervosa include the following EXCEPT:
a) It is seen predominantly in females, rarely in males.
b) Most affected patients have an obsessive -compulsive personality.
c) Mean 24-hour concentration of cortisol is twice normal.
d) Thyroid hormones are in the normal range.
e) Occasionally fatal

95-Amenorrhea could be due to which of the following EXCEPT:


a) Combined oral contraceptive.
b) Depot medroxy progesterone acetate.
c) GnRH analogues.
d) Danazol.
e) Bromocriptine.

96-Which of the following conditions is suitable for clomiphene citrate stimulation?


a) Gonadal dysgenesis
b) Asherman's syndrome
c) Resistant ovary syndrome
d) Polycystic ovarian syndrome
e) Kallmann's syndrome

97-Cryptomenorrhea may present with the following EXCEPT:


a) Acute urinary retention.
b) Hematocolpos
c) Lower abdominal pain.
d) Premenstrual spotting.
e) Cyclic pain

Directions (Questions 98 through 110): Each set of matching questions in this section
consists of a list of 5 to 6 lettered options followed by several numbered items. For each
item, select the ONE best lettered option that is most closely associated with it. Each
lettered heading may be selected once, more than once, or not at all.

Questions 103-107
Link the following syndromes to their corresponding presentations
a) Laurence Moon Biedl.
b) Kallmann syndrome.
c) Sheehan syndrome.
d) Savage syndrome
e) Asherman syndrome.
f) Swyer syndrome

103- Failure of lactation.


104- Post curettage amenorrhea.
105- Amenorrhea & anosmia.
106- Retinitis pigmentosa
107- Resistant ovary syndrome

true/false

111- As regards luteinizing hormone, the following is (are) true


1) Its plasma levels are increased throughout pregnancy.
2) Its plasma level is increased in postmenopausal women.
3) It is bound to plasma protein.
4) It stimulates the synthesis of testosterone in the male.
5) Its release is stimulated by thyrotropin releasing hormone.
1) It stimulates androgen production.
2) Its plasma concentration is increased in pregnancy.
3) Its plasma concentrations are increased in ovarian failure.
4) It is a steroid hormone.
5) It is released at a constant rate throughout the menstrual cycle.

113- As regards follicle stimulating hormone, the following is (are) true:


1) It stimulates spermatogenesis.
2) Its plasma concentration is high in Klinefelter syndrome.
3) It stimulates ovarian estrogen production.
4) It is produced in the hypothalamus.
5) It prevents regression of the corpus luteum.

Pituitary follicle stimulating hormone is:


1 ) A glycoprotein.
2) Excreted in increased amount at menopause.
3) Excreted by the male.
4) Secreted by pars intermedia of the hypophysis cerebri.
5) Identical with human chorionic gonadotrophin.

114- As regards ovulation in the human the following is (are) true:


1) Is associated with a surge of luteinizing hormone.
2) Is characteristically followed by the development of secretory endometrium.
3) Is associated with an increase in motility of the fallopian tube.
4) Is associated with a sustained fall in basal body temperature.
5) Is followed by a rise in urinary pregnanetriol.

115- As regards estrogen, the following is (are) true:


1) Cannot be detected in the blood of postmenopausal women.
2) Is produced in the corpus luteum.
3) Is mainly secreted by the ovary as estrone.
4) Is the dominant gonadal hormone at puberty.
5) Is responsible for secretory changes in the endometrium.

116- The anterior pituitary:


1) 2) Produces vasopressin.
3) Is controlled by releasing factors produced in the hypothalamus.
4) 5) Lies above the optic chiasma.

117- In a normal human menstrual cycle the corpus luteum:


1) Remains active for 3-4 weeks.
2) Secretes progesterone.
3) Is maintained by human chorionic gonadotrophin.
4) Secretes estrogen.
5) Secretes pregnanediol.
118-

119- In the first half of a normal menstrual cycle:


1) Mitotic figures are seen in the endometrium.
2) Serum progesterone levels are high.
3) Some ovarian follicles degenerate.
4) The endometrium is rich in glycogen.
5) The corpus luteum begins to degenerate.

120- In normal puberty, the following features are characteristics:


1) Pubic hair growth is the first sign.
2) The first menstrual cycles are anovulatory.
3) Axillary hair growth occurs after the first menstrual period.
4) Changes in the vaginal epithelium.
5) Cessation of growth.

121- The following hormonal changes occur after menopause:


1) Estrogen levels decrease dramatically.
2) There is a relative increase in the testosterone level.
3) The main source of progesterone production is from the adrenal glands.
4) Estradiol accounts for most of circulating estrogen which results mainly from
peripheral conversion of androstenedione.
5) Gonadotrophins levels show no changes.

122- The following are hormonal changes associated with menopause:


2) Increased FSH.
3) Increased androstenedione.
4) Increased SHBG.
5) Increased LH.
123- Unopposed estrogen therapy for the postmenopausal woman:
1) Improves the urethral syndrome.
2) Decreases urinary calcium excretion.
3) Causes increased incidence of endometrial carcinoma.
4) Causes decreased incidence of myocardial infraction.
5) Causes hypertension.
124- Premature menopause in genotypically female could be due to:
1) Turner's syndrome.
2) Use of OCs for long time.
3) Excess exposure to radiation.
4) Klinefelter syndrome.
5) Testicular feminization syndrome.

125- The following statements about osteoporosis are correct:


1) There is increased osteoclastic activity resulted in fragile bone.
2) Is prevented by estrogens.
3) Bisphosphonates are the most effective bone building drugs.
4) Diagnosed mainly by determination of serum calcium.
5) Occurs more commonly in patients with Turner's syndrome.

126- Risk factors for development of osteoporosis include the following:


1) Cigarette smoking.
2) Long term glucocorticoid therapy.
3) Premature ovarian failure.
4) Is more common in black than white women.
5) Obese women.

127- Absolute contraindication for HRT:


1) Active liver disease.
2) Hypertension.
3) Recent history of thrombo-embolic attack.
4) Gall bladder disease.
5) Diabetes mellitus.

128- Hirsutism:
1) Adrenal enzyme deficiency can present in adult women.
2) The oral contraceptive pill can be used to treat hirsutism.
3) Cyproterone acetate should be given in a contraceptive regimen.
4) Dexamethasone is an alternative anti-androgen.
5) Circulating testosterone is always elevated in hirsute women.

129- Suggestions of anovulation include:


1)
2) Disturbed menstrual cycles.
3) Serial ultrasound scans.
4) Secretory changes detected at histological examination of the endometrium.
5) Biphasic basal body temperature chart.

130- Clomiphene citrate:


1) Has antiestrogenic effects.
2) Side effects include multiple pregnancy.
3) Ovarian hyperstimulation syndrome is rare.
4) Is a steroidal agent.
5) Is a natural product.
1) Causes breast carcinoma.
2) Is associated with ovarian cyst formation.
3) Increases the risk of fetal malformations.
4) Causes vasomotor symptoms as a side effect.
5) Is prescribed in a starting dose of 500 mg daily for five days/month.

132- Drugs of antiestrogenic effects include:


1) Tamoxifen.
2) LHRH analogues.
3) Clomiphene citrate.
4) Mifepristone.
5) Bromocriptine.

133- In PCOD:
1) PCOD is the commonest cause of anovulatory infertility.
2) Pregnant women with PCOD have an increased risk of gestational diabetes.
3) Anovulatory women with PCOD are at increased risk of endometrial cancer.
4) Hyperlipidemia is a cause of PCOD.
5) The majority of anovulatory women with PCOD are estrogen deficient.

134- Dysfunctional uterine bleeding is associated with:


1) Metropathia hemorrhagica.
2) Abnormal hormone profiles in more than 50% of patients.
3) Increased levels of PGE2 in the endometrium.
4) Chronic pelvic inflammatory disease.
5) Estrogens inhibiting the arachidonic acid cascade.
135- A woman presenting with cyclical menorrhagia at the age of 46 years:
1) Is likely to be cured by an oral progestogen.
2) Is likely to be cured by hormone replacement therapy (HRT).
3) may require hysterectomy if medical treatment fails.
4) Should be treated by dilatation and curettage.
5) Has an 80% chance of having anovulatory cycles.

136- Patients with testicular feminization characteristically:


1) Are 46, XY.
2) Have a hypoplastic uterus.
3) Have fairly developed breasts.
4) Are chromatin positive.
5) Have secondary amenorrhea.

137- In abnormal uterine bleeding in adolescents:


1) In girls with severe menorrhagia coagulation disorder should be excluded.
2) Ovulatory dysfunction is a rare cause.
3) Pregnancy complications should be excluded.
4) The commonest hematological disorder is von Willebrand's disease.
5) Evaluation of the patient must include a pelvic examination.

138- Turner's syndrome:


1) Is associated with neonatal edema of the feet.
2) Is the commonest chromosomal abnormality in aborted fetuses.
3) Babies display neonatal sneezing.
4) Has ventricular septal defect as the commonest cardiac abnormality.
5) Is autosomal dominant.

139- Bromocriptine:
1) Is a dopamine agonist.
2) Inhibits prolactin secretion.
3) Can be administered vaginally.
4) Is more potent than lisuride drug in treatment of hyperprolactinemia.
5) Can cause hypertension.

140- Prolactin:
a)
b) Bromocriptine is the drug of choice in hyperprolactinemia.
c) Is secreted from the posterior pituitary gland.
d) Approximately one -third of patients with a micro-adenoma undergo spontaneous
resolution.
e) Bromocriptine must be stopped when pregnant.
141- In luteinized unruptured follicle:
a) Progesterone is secreted normally.
b) Menstrual cycles are usually regular.
c) Antiprostaglandins are needed for rupture of ovarian follicles.
d) Induction of ovulation is a useful modality for treatment.
e) Patient can benefit from human chorionic gonadotrophin (hCG) injections given in
high doses at mid-luteal phase.

142- Gonadotrophin releasing hormone (GnRH):


a) Is used for induction of ovulation in patients with hypergonadotrophic
hypogonadism.
b) Is indicated in treatment of infertility associated with Kallmann's syndrome.
c) Must be delivered continuously for 5-6 days to reach follicular maturity.
d) Has a similar incidence of multiple pregnancy rate as with human menopausal
gonadotrophin therapy (HMG).
e) Risk of ovarian hyperstimulation is absent.

143- Human menopausal gonadotrophin (HMG):


a) Are extracted from postmenopausal blood.
b) Can be given intramuscular or intravenous.
c) May result in ovarian hyperstimulation.
d) Requires human chorionic gonadotrophins (hCG) to be given after its use to induce
ovulation.
e) Can be monitored by serial measurement of serum progesterone levels.
GYNECOLOGICAL DIAGNOSIS

Directions: Each of the following numbered items or incomplete statements in this


section is followed by numbered items that in certain combination leads to the BEST
lettered combined answer or/and completion. According to the following directions,
choose the BEST lettered answer:
If
1,23 are correct, choose A
1,3 are correct, choose B
2,4 are correct, choose C
4 is correct, choose D
All are correct, choose E

144- Rectal examination in the gynecological practice:


1) Is useful in the diagnosis of enterocele.
2) Is useful in suspected pelvic abscess.
3) Is indicated in the assessment of carcinoma of the cervix.
4) Is to be preformed as a routine in the gynecological practice.
5) Is useful in assessment of ectopic pregnancy.
GENITAL DISPLACEMENT

Directions: Each of the numbered items or incomplete statements in this section is


followed by answers or by completions of the statement. Select the ONE lettered answer
or completion that is BEST in each case.

145- As regards cystocele the following is true EXCEPT:


a) Is a prolapse of the bladder in upper part of anterior vaginal wall.
b) Is common after the menopause.
c) Is the cause of stress incontinence of urine.
d) May lead to urinary infection.
e) Is very uncommon in nulliparous women.

146- Genital prolapse is associated with the following EXCEPT:


a) Multiparity.
b) Prolonged second stage of labor.
c) Unrepaired hidden perineal tear.
d) Improperly applied forceps delivery.
e) Negroes' race.

147- Factors important in the development of genital prolapse include the following
EXCEPT::
a) Poor tissue strength.
b) c) Chronic straining at bowel movements.
d) Menopause.
e) Childbirth trauma.
148-
149-

150- As regards enterocele, the following is correct:


a) It is a prolapse of the rectum.
b) It may occur following colposuspension.
c) Sigmoidoscopy is used for diagnosis
d) It may resolve spontaneously.
e) It is a common cause of stress incontinence.
a) It is not a true hernia.
b) It is a herniation of the bladder floor into the vagina,
c) It is a prolapse of the uterus and vaginal wall outside the body.
d) It is a protrusion of the pelvic peritoneal sac and vaginal wall into the vagina.
e) It is a herniation of the rectal and vaginal wall into the vagina.

152- As regards anterior colporrhaphy the following is true EXCEPT:


a) May cause temporary retention of urine.
b) Is the treatment of midline defect of anterior vaginal wall prolapse
c) Is frequently combined with vaginal hysterectomy.
d) Better avoided in patients with urge incontinence.
e) Should be carried out only after completion of childbearing.
Directions (Questions 153 through 157): Each set of matching questions in this section
consists of a list of 6 lettered options followed by several numbered items. For each item,
select the ONE best lettered option that is most closely associated with it. Each lettered
heading may be selected once, more than once, or not at all. Match the most suitable
operation with the disease entity encountered
a) Sacrospinous colpopexy
b) Anterior colporrhaphy
c) Burch colposuspension
d) Le Forte's operation
e) Fothergill's operation
f) Sacral cervicopexy

153- Second degree uterine descent in 32-year-old woman wants to preserve her
potential fertility
154- Second degree uterine descent in 75-year-old-woman
155- Stress (urodynamic) incontinence
156-Vault prolapse
157- Cystocele with midline defect

Directions: Each of the following numbered items or incomplete statements in this


section is followed by numbered items that in certain combination leads to the BEST
lettered combined answer or/and completion. According to the following directions,
choose the BEST lettered answer:
If
1,2,3 are correct, choose A
1,3 are correct, choose B
2,4 are correct, choose C
4 is correct, choose D
AH are correct, choose E

158- Uterovaginal prolapse:


1) The condition is worse in the erect position.
2) May cause intestinal obstruction if there is a large rectocele.
3) The cervix is often elongated.
4)

159- Retroversion of the uterus:


1) Is a common cause of infertility.
2)
3) May be corrected by a fothergill operation.
4) Occurs in 20% of normal women.
5) Is caused by heavy lifting.

160- Genital prolapse-etiological factors include:


1) Multiparity.
2) Failed postnatal exercises.
3) Crede's maneuver.
4) First degree tears.
5)

161- Second degree uterine prolapse:


1) is diagnosed when the cervix protrudes through the vulval orifice.
2) Is also known as complete procidentia.
3)

162- Structures which prevent prolapse of the uterus and vagina include:
1) Levator ani muscle.
2) Uterosacral ligaments.
3) Cardinal ligaments.
4) Round ligaments.
5) Broad ligaments.

163- Factors predisposing to the development of uterovaginal prolapse include:


1) Congenital weakness of the supporting ligaments.
2) Postmenopausal atrophy.
3) Injury during childbirth.
4) Endometrial polyps.
5) Ovarian tumors.

164- The Manchester repair includes:


1) Amputation of the cervix.
2) Posterior colpoperineorrhaphy.
3) Anterior colporrhaphy.
4) Vaginal hysterectomy.
5)

Directions: In the following questions, each item has 5 lettered options each of which
could be a true or a false statement. Tick each of these options according to whether
they are false or true

165- An enterocele:
a) Usually contains rectum.
b) In lined by peritoneum.
c) Frequently contains small bowel.
d) May be a long-term complication of vaginal hysterectomy.
e)

166- A cystocele:
a) Is usually associated with stress incontinence.
b) Contains bladder.
c) May present as urinary retention.
d) Is treated by Fothergill’s operation.
e) Is best treated with a ring pessary.

167- Backache due to gynecological causes:


a) May be caused by a mobile retroverted uterus.
b) Is usually felt in the lumbar area.
c) May be caused by endometriosis.
d) May be due to uterine prolapse.
e) May be due to chronic pelvic infection.
GENITAL TRACT INFECTIONS
Directions: Each of the numbered items or incomplete statements in this section is
followed by answers or by completions of the statement. Select the ONE lettered answer
or completion that is BEST in each case.
168- The following about Mycoplasma are correct EXCEPT:
a) Causes male infertility.
b) Is gram-negative.
c) Can be successfully treated with penicillin.
d) Causes neonatal pneumonia.
e) Causes pelvic inflammatory disease.

169- The natural defense of the vagina to infection include all of the following
EXCEPT:
a) The vaginal pH.
b) The presence of Doderlein's bacilli.
c) The physical apposition of the pudendal cleft and the vaginal walls.
d) The bacteriostatic secretions of vaginal glands.
e) The vaginal stratified squamous epithelium.
170- As regards Gardnerella vaginalis vaginitis all of the following are correct
EXCEPT:
a) It is usually asymptomatic.
b) It may present with a foul smell discharge.
e) It may progress to acute pelvic inflammatory disease.
d) Its discharge may increase after intercourse.
e) It can be diagnosed by clue cells on Gram staining of vaginal discharge.

171- The following about Chlamydia trachomatis are correct EXCEPT::


a) Can be cultured from vaginal discharge.
b) Is a common cause of vaginal discharge.
c) May be treated with either erythromycin or aminoglycosides.
d) Can cause neonatal pneumonia.
e) Can cause sterile pyuria.
172- The single-dose parenteral drug of choice used in uncomplicated gonorrhea is:
a) Ceftriaxone 250 mg
b) Cefixime 400 mg
c) Ciprofloxacin 500 mg
d) Ofloxacin 400 mg
e) Spectinomycin 500 mg
173- The following about human papilloma virus (HPV) infection are correct EXCEPT:
a) It is the most common viral STDs.
b) It may lead to CIN and cervical cancer.
c) It is due to RNA virus
d) Infection may be warty or flat condyloma.
e) Infection is usually associated with others STDs.
174- The commonest site to be affected by genital tuberculosis is:
a) The ovaries
b) The fallopian tubes
c) The uterus
d) The cervix
e) The vagina

175- The following about bacterial vaginosis are correct EXCEPT:


a) It is the commonest vaginal infection.
b) Vaginal PH is usually < 4.5.
c) The discharge cause considerable itching.
d) Cure may be achieved by oral metronidazole.
e) The organism is a bacterium.

176- The following about Candida! infection are correct EXCEPT:


a) The infection is common with pregnancy
b) Vaginal PH is usually alkaline.
c) Vulval itching may occur.
d) Vaginal isoconazole or miconazole are effective.
e) The organism is yeast-like.

177- The following about Trichomonas vaginalis (T.V.) are correct EXCEPT:
a) T.V may be a sexually transmitted disease
b) Most patients having TV have symptoms.
c) The disease may cause itching.
d) Unlike bacterial vaginosis, metronidazole is ineffective therapy.
e) It may cause a flea-bitten appearance of the cervix

178- Regarding syphilis, all of the following are correct EXCEPT:


a) The organism is diagnosed by dark field illumination.
b) The classic finding in primary syphilis is a hard chancre.
c) Secondary syphilis is associated with a rash over the hands and feet.
d) Gummas are found in late syphilis.
e) Syphilis is produced by a spirochete.

Directions (Questions 179 through 199): Each set of matching questions in this section
consists of a list of 2 to 8 lettered options followed by several numbered items. For each
item, select the ONE best lettered option that is most closely associated with it. Each
lettered heading may be selected once, more than once, or not at all.

Questions 179-183
Match each condition to the appropriate pH
a) Generally acidic
b) Generally alkaline

179- Cervical secretions


180- Semen
181- Urine
182- Menstruation
183- Normal vaginal pH

Questions 184-188
Match the effect of each secretion
a) Turns pH of the vagina more acidic
b) Turns pH of the vagina more alkaline
c) Keeps pH of the vagina constant

184- Menopause
185- Lactobacilli
186- Semen
187- Menstruation
188- Estrogen therapy

Questions 189-193
Match the genital organism with the most appropriate chemotherapeutic
agent:
a) Azithromycin
b) Spectinomycin
c) Itraconazole
d) Penicillin G
e) Tetracycline
f) Clindamycin
g) Metronidazole

189- Neisseria gonorrheae (Uncomplicated)


190- Chlamydia trachomatis
191- Gardnerella vagina I is
192- Candida albicans
193- Trichomonas vaginalis

Questions 194-199
Match the Genital infection with the diagnostic modality
a) Culture on Thayer Martin medium
b) Culture on Dorset egg medium
c) Culture on Sabouraud medium
d) Culture on aerobic blood agar
e) Micro-immunofluorescent test
f) Western blot
g) PCR and Southern blot.
h) Whiff test

194- Chlamydia infection


195- Candida! infection
196- Gonorrhea infection
197- Gardenerella vaginalis
198- HPV
199- HIV
Directions: Each of the following numbered items or incomplete statements in this
section is followed by numbered items that in certain combination leads to the BEST
lettered combined answer or/and completion. According to the following directions,
choose the BEST lettered answer:
If
1,2,3 are correct, choose A
1,3 are correct, choose B
2,4 are correct, choose C
1,2,3,4 are correct, choose D
All are correct, choose E

200- The following about vaginal discharge are correct:


1) Leucorrhea, means excessive amount of normal vaginal discharge.
2) Cervicitis is inflamed columnar epithelium on the cervix.
3) Atrophic vaginitis is relatively common in postmenopausal women.
4) Progesterone causes a proliferation of the vaginal epithelium.
5) The amount of vaginal discharge increases only during infection.

201- As regards chlamydial infection, the following are correct


1) The organisms are obligatory intracellular organisms.
2) It may cause sterile pyuria.
3) Chlamydia and gonorrhea are the main causes of PID.
4) The husband should be treated.
5) Azithromycin 1 gm in single dose is an effective treatment.

202- Tuberculosis of Genital Tract:


1) The tubes are the commonest site.
2) Ft is detected in 5% of infertile patients.
3) The treatment is mainly medical.
4) Menstrual disorders are common.
5) The infection is usually sexually transmitted

203- The following bacterial infections may be sexually-transmitted


1) Neisseria gonococci.
2) Gardnerella vaginalis.
3) Treponema palladium.
4) Trichomonas vaginalis.
5) Group B streptococci.

204- Herpes Simplex:


1) Type I does not invade the genital organs.
2) In primary forms, the leading complaint is agonizing pain.
3) The recurrent attacks tend to be more severe and have generalized manifestations
more than the primary attack.
4) During primary infection vaginal delivery may result in neonatal herpes in about
40%.
5) Long-term suppression with acyclovir increases the number of recurrent attacks.

Directions: In the following questions, each item has 5 lettered options each of which
could be a true or a false statement. Tick each of these options according to whether
they are false or true

205- The following about human Papilloma Virus is (are) true:


a) It is a rare type of STD.
b) It is classified into subtypes according to types of antibodies.
c) Has a definite pathological role in development of CIN.
d) Koilocytic atypia is the main feature in the cytological study.
e) The most effective anti-viral is acyclovir.
206- The following about pelvic tuberculosis is (are) true:
a) Is most often bovine.
b) Is post primary.
c) Is associated with amenorrhea.
d) More often involves the fallopian tubes.
e) Is decreasing in incidence.

207- The following about genital tuberculosis is (are) true:


a) The most affected site is the fallopian tube.
b) The main route of transmission is through sexual intercourse.
c) Infertility is the leading complaint.
d) Negative tuberculin test excludes the presence of the disease.
e) Hysterectomy + BSO is the main therapeutic option.
TRAUMA TO THE FEMALE GENITAL TRACT

Directions: Each of the numbered items or incomplete statements in this section is


followed by answers or by completions of the statement. Select the ONE lettered answer
or completion that is BEST in each case.

208- Rectovaginal fistulas result from all of the following EXCEPT:


a) Obstetric delivery.
b) Irradiation to the pelvis.
c) Carcinoma.
d) Hemorrhoidectomy.
e) Herpes vulvitis.
GYNECOLOGIC UROLOGY
Directions: Each of the numbered items or incomplete statements in this section is
followed by answers or by completions of the statement. Select the ONE lettered answer
or completion that is BEST in each case.

209- True incontinence occurs in all of the following EXCEPT:


a) Vesicovaginal fistula
b) Bilateral ureterovaginal fistula
c) Unilateral ureterovaginal fistula
d) Vesico-cervicovaginal fistula
e) Vesico-urethrovaginal fistula

210- The most important suspensory mechanism of the urethra is (are):


a) Anterior pubourethral ligaments
b) Intermediate pubourethral ligaments
c) Posterior pubourethral ligaments
d) White line
e) Pubocervical fascia

211- All the following about genuine stress incontinence in the female are correct
EXCEPT:
a) It occurs transiently during pregnancy.
b) It is more common in parous women.
c) It can be corrected surgically.
d) It is associated with a high urethral pressure profile amplitude on urodynamic studies
e) It is usually associated with utero-vaginal prolapse.

212- The following are treatment modalities for overactive bladder EXCEPT:
a) Parasympathomimetics
b) Smooth muscle relaxants
c) Anticholinergics.
d) Behavioral therapy
e) Tricyclic anti depressants

213- Utero-vesical fistula usually presents with:


a) Amenorrhea
b) Menuria
c) Dysmenorrhea
d) Terminal hematuria
e) True incontinence

Directions (Questions 214 through 223): Each set of matching questions in this section
consists of a list of 4 to 6 lettered options followed by several numbered items. For each
item, select the ONE best lettered option that is most closely associated with it. Each
lettered heading may be selected once, more than once, or not at all.
Questions 214-218
Match the common presentation with the disease entity
a) True urinary incontinence
b) Menuria
c) Urge incontinence
d) Partial urinary incontinence
e) Overflow urinary incontinence
f) Urinary incontinence at sexual penetration
214- Vesicovaginal fistula
215- Uterovesical fistula
216- Left ureterovaginal fistula
217- Genuine stress incontinence
218- Overactive bladder
Questions 219-223
Match the operation with the disease entity
a) Urodynamic stress incontinence
b) Overactive bladder
c) Vesicovaginal fistula
d) Vault vesical fistula
219- Sims dedoublement operation
220- Burch colposuspension
221- Marshal Marchetti Krantz operation
222- Clam cystoplasty
223- Latzko operation (partial colpocleisis)
Directions: Each of the following numbered items or incomplete statements in this
section is followed by numbered items that in certain combination leads to the BEST
lettered combined answer or/and completion. According to the following directions,
choose the BEST lettered answer:
If
1,2,3 are correct, choose A
1,3 are correct, choose B
2,4 are correct, choose C
1,2,3,4 are correct, choose D
All are correct, choose E
224- In the treatment of genuine stress incontinence (GSI):
1) Preoperative urethral pressure profilometry can predict the success of surgical
treatment.
2) Patients with co-existent detrusor instability could have surgical treatment.
3) Vaginal surgery for stress incontinence carries a high failure rate.
4) The standard operation is the suburethral sling
5) Physiotherapy will improve symptoms in about 20% of patients.
225- Urethral caruncle:
1) Usually covered by transitional epithelium.
2) Usually asymptomatic.
3) May resemble urethral carcinoma.
4) Usually originates from the anterior wall of the urethra.
5) Occurs in children.
BREAST DISEASES

Directions: Each of the numbered items or incomplete statements in this section is


followed by answers or by completions of the statement. Select the ONE lettered answer
or completion that is BEST in each case.

226- The following could be a treatment for mastalgia EXCEPT:


a) Surgery.
b) Bromocriptine.
c) Danazol
d) Gammalenolenic acid (GLA)
e) Antiprostaglandins

227- The following about nipple discharge are correct EXCEPT:


a) It is common complaint.
b) Galactorrhea is atypically bilateral.
c) It is usually physiologic.
d) Duct ectasia is a benign cause for the discharge.
e) Mammogram should be a part of the evaluation with bloody discharge.
BENIGN GYNECOLOGIC DISEASES

Directions: Each of the numbered items or incomplete statements in this section is


followed by answers or by completions of the statement. Select the ONE lettered answer
or completion that is BEST in each case.

228- Regarding cervical intraepithelial neoplasia (CIN), all the following are correct
EXCEPT:
a) Transformation zone is the most susceptible location for development.
b) It is usually unifocal.
c) Abnormal vascular pattern seen by colposcopy include punctuation, mosaicism and
acetowhite area.
d) Total hysterectomy is a line of management.
e) Cervical conization is best done by loop electrosurgical excision procedure

229- Regarding cervical polyps all the following are correct EXCEPT:
a) May cause post coital bleeding.
b) The most common type is mucous polyps.
c) Usually they arise from the cervical canal.
d) They are usually pre cancerous.
e) Cervical polyps should be subjected to histopathological examination.

230- AH the following about Bartholin's glands are correct EXCEPT:


a) Are remnants of mesonephric origin.
b) Can commonly be infected by E coli.
c) Excision of the cyst when infected is usually contraindicated.
d) Are situated at the posterior parts of labia majora.
e) Should be marsupialized when acutely infected.

231- The commonest secondary change in uterine fibroids is:


a) Fatty degeneration
b) Myxomatous degeneration
c) Hyaline degeneration
d) Cystic degeneration
e) Calcification

232- The most common site of uterine myoma is:


a) Subserous
b) Interstitial
c) Subserous
d) Cornual
e) Cervical

233- The following are risk factors to developing uterine myomata EXCEPT;
a) Low parity
b) Late menarche
c) Negroid race
d) Positive family history of the disease
e) Associated endometriosis
234- Sarcomatous change in uterine fibroids occurs in:
a) 0.05%.
b) 0.5%
c) 1.5%
d) 5%
e) 15%

235- All the following about uterine fibroids are correct EXCEPT:
a) Occur in over 30% of women of reproductive age.
b) Can undergo sarcomatous change in 5% of cases.
c) Are associated with menorrhagia.
d) Can cause polycythemia.
e) Can cause pressure manifestations in the pelvis.

236- Intramural fibroids mainly cause:


a) Intermenstrual bleeding.
b) Postcoital bleeding.
c) Postmenopausal bleeding.
d) Deep dyspareunia.
e) Menorrhagia.

237- Endometriosis characteristically occurs in women who are:


a) In lower socio-economic group.
b) Perimenopausal.
c) Nulliparous.
d) Using IUCD.
e) Using Oral contraceptives.

238- All the following about uterine fibroids are correct EXCEPT:
a) Are estrogen dependent.
b) Get smaller during treatment with progestogens.
c) Are usually asymptomatic.
d) Shrink in response to treatment with LHRH agonists.
e) May be treated conservatively

239- The etiology of endometriosis relates to the following EXCEPT:


a) A transformation of celomic epithelium.
b) Direct invasion through the uterine serosa into the pelvic capacity.
c) Retrograde menstrual flow.
d) Vascular transport of endometrial fragments.
e) Lymphatic transport of endometrial fragments.

240- The following physical findings are consistent with a diagnosis of endometriosis
EXCEPT:
a) Fixed retroversion of the uterus.
b) Tender pelvic masses.
c) Adnexal enlargement.
d) Cul-de-sac nodules.
e) Omental nodules

241- A woman with symptomatic endometriosis is likely to have the following


complaints EXCEPT:
a) dyspareunia
b) mood swings
c) painful defecation
d) severe dysmenorrhea
e) infertility

242- Endometriosis treated with prolonged estrogen and progesterone combination


therapy exhibits which of the following histological characteristics?
a) Marked edema
b) Decidual-like reaction
c) Glandular hypertrophy
d) Inflammatory infiltrate
e) Cyclic changes

243- The major symptom(s) of adenomyosis is/are


a) irregular uterine enlargement
b) menorrhagia and dysmenorrhea
c) urinary frequency
d) dysmenorrhea and infertility
e) pressure symptoms

244- The Following are effective in the treatment of endometriosis EXCEPT:


a) Clomiphene.
b) Oophorectomy.
c) Hysterectomy or Depo-provera.
d) Oral contraceptive.
e) Danazol.

245- Retrograde menstruation is the most accepted cause of endometriosis. The


following statements may be cited as evidence for this theory EXCEPT:
a) Inversion of the cervix of a monkey into the peritoneal cavity can cause
endometriosis.
b) Endometrial tissue can be cultured successfully.
c) Menstrual blood can come from the ends of the fallopian tubes of some women.
d) Endometrial glands can arise from celomic epithelium.
e) The disease is often found in the dependent portion of the pelvis.

246- Non-neoplastic cysts of the ovary include the following EXCEPT;


a) Theca-lutein cysts.
b) Pregnancy luteomas.
c) Endometriotic cysts.
d) Corpus luteum cysts.
e) Dermoid cyst
247- A 54-year-old woman is found to have endometrial hyperplasia on endometrial
biopsy. A functional ovarian tumor to be suspected is a:
a) Lipid cell tumor.
b) Granulosa-theca cell tumor.
c) Sertoli-Leydig tumor.
d) Mutinous cystadenocarcinoma.
e) Polycystic ovary.
248- Treatment of benign ovarian neoplasms may include the following EXCEPT:
a) Debulking.
b) Ovarian cystectomy.
c) Unilateral oophorectomy.
d) Hysterectomy plus unilateral salpingo-oophorectomy
e) Pan hysterectomy.

249- In contrast to a malignant ovarian tumor, a benign tumor has which of the
following gross features?
a) Excrescences on the surface.
b) Peritoneal implants.
c) Intra-cystic papillae.
d) Free mobility.
e) Capsule rupture.

250- The most common benign mass of the cervix and endocervix is a:
a) polyp
b) leiomyoma
c) Nabothian cyst
d) Cervical hood
e) Gartner's duct cyst

251- A 37-year-old woman complains of postcoital bleeding. The LEAST likely cause of
her bleeding would be cervical
a) polyps
b) ectropion
c) carcinoma
d) Nabothian cyst
e) infection

252- All the following about uterine fibroids are correct EXCEPT:
a) They originate from smooth muscle.
b) They contain muscle and connective tissues.
c) They are usually multiple.
d) They are visually submucous.
e) They are uncommon in the cervix.

Directions: Each of the following numbered items or incomplete statements in this


section is followed by numbered items that in certain combination leads to the BEST
lettered combined answer or/and completion. According to the following directions,
choose the BEST lettered answer:
If
1,2,3 are correct, choose A
1,3 are correct, choose B
2,4 are correct, choose C
1,23,4 are correct, choose D
All are correct, choose E
253- The complications of benign ovarian neoplasms include
1) Torsion.
2) Infection.
3) Malignant change.
4) Rupture.
5) Incarceration.
254- Regarding the Bartholin cyst, the following are correct:
1) It is the commonest cyst in the vulva.
2) True cystic swelling is present in anterior part of labium major.
3) The main cause is obstruction of Bartholin gland duct by chronic infection.
4) Should be excised if the patient is above 30 years.
5) The main symptom is throbbing pain.
255- As regard purities vulva the following are correct:
1) Pre malignant lesions are one of the causes.
2) Malnutrition & vitamins deficiencies may lead to it.
3) Pruritus vulvae resistant to treatment should be subjected to biopsy.
4) Estrogen can be used for treatment.
5) Surgical treatment includes circum-vulval incision, or simple vulvectomy.
256- As regards cervical ectopy (erosion), the following are correct:
1) It is an ulcer of the cervix.
2) It is treated conservatively in pregnant females.
3) Cauterization should be done as first line of therapy.
4) Pap smear is advisable before management.
5) Commonly cause pain, dyspareunia & low back pain.
257- Regarding cervical ectropion:
1) It can be differentiated from cervical lesion by presence of cervical tear.
2) It may associates incompetent cervix.
3) It is treated by trachelorrhaphy if cervical tear is present.
4) Endo-cervix appears due to ulceration.
5) It can be treated by cauterization.
258- Fibroids can result in:
1) Menorrhagia.
2) Polycythemia.
3) Constipation.
4) Acute urinary retention.
5) Infertility.
259- In pelvic endometriosis:
1) Treatment by danazol may produce hirsutism.
2) The tubes are uncommonly blocked.
3) There is a close association with the unruptured follicle syndrome.
4) The amount of pelvic pain is related to the extent of the disease.
5) Clomiphene citrate should be used to induce ovulation.

260- Characteristic symptoms of endometriosis include:


1) Dysmenorrhea.
2) Superficial dyspareunia.
3) Infertility.
4) Amenorrhea.
5) Pre-menstrual tension.

261- Functional ovarian masses include:


1) Follicular cysts.
2) Endometriomas.
3) Luteomas.
4) Dermoid cysts.
5) Fibromas.

262- The following is essential in diagnosis of pelvic endometriosis:


1) Laparoscopy.
2) CA-125.
3) Biopsy form the suspicious nodules.
4) Ultrasonography.
5) Hysteroscopy.

Directions: In the following questions, each item has 5 lettered options each of which
could be a true or a false statement. Tick each of these options according to whether
they are false or true

263- Benign cystic teratoma (dermoid cyst):


a) It represents 50% of the ovarian neoplasms.
b) It is the commonest ovarian tumor during pregnancy.
c) Malignant change occurs in about 1.5%.
d) Choriocarcinoma is the commonest malignant change.
e) Rarely causes harm to the patient when ruptures.

264- Dermoid cysts:


a) Are germ cell tumors.
b) Are bilateral in 40-60% of cases.
c) Are the commonest neoplastic cysts detected during pregnancy.
d) Are malignant in 10% of cases.
e) Are frequently XY.

265- Brenner's tumor


a) Usually develops in patients over 40 years, h) Usually is bilateral.
c) Rarely turns malignant.
d) May secrete androgen.
e) Usually found in the wall of mutinous cystadenoma.

266- Dermoid cysts:


a) Represent 20% of all ovarian neoplasms.
b) Usually are unilateral.
c) Are the commonest ovarian tumors found during pregnancy.
d) May contain tissues of ectodermal origin.
e) 15% of cases turn malignant.
GYNECOLOGIC ONCOLOGY
Directions: Each of the numbered items or incomplete statements in this section is
followed by answers or by completions of the statement. Select the ONE lettered answer
or completion that is BEST in each case.

267- High risk factors of CIN include the following EXCEPT:


a) HPV types 6, 11.
b) HIV is a predisposing factor for CIN.
c) Smoking
d) First sexual intercourse before 18 years age
e) Multiple sexual partners

268- CIN III lesions extending into the cervical canal are:
a) Often invasive.
b) Treated by radiotherapy.
c) Treated by hysterectomy.
d) Safely treated by cone biopsy.
e) Safely treated by large loop excision of the transformation zone.

269- The following statements are true of large loop excision of the cervical
transformation zone (LLETZ) EXCEPT;
a) It is used even when a CO2 laser is available.
b) It requires anesthesia.
c) It is a cheap and effective way of removing the cervical transformation zone.
d) Secondary hemorrhage is a rare complication.
e) Cervical stenosis may occur.

270- Endometrial hyperplasia could be expected in the following conditions:


a) Endodermal sinus tumor.
b) Cystic teratoma.
c) Polycystic ovary disease.
d) Sertoli-Leydig cell tumor.
e) Dysgerminoma.
271- Malignant melanoma represents what percent of the whole cancer vulva
occurring?
a) 1%
b) 3%
c) 5%
d) 10%
e) 15%
272- The colposcopic pattern of CIN III is characterized by the following EXCEPT:
a) Surface regularity of the squamous epithelium.
b) A marked acetowhite appearance.
c) Coarse epithelial punctation.
d) A transformation zone that bleeds to touch.
e) Multisector involvement of the transformation zone.
273- Complex endometrial hyperplasia without atypia in a woman aged 41 years could
be treated with the following EXCEPT:
a) Progestins
b) Estrogens
c) Progestins plus estrogens
d) Oral contraceptive pills
e) Hysterectomy
274- Carcinoma of the vulva:
a) usually ulcerates even if not advanced.
b) Is usually histologically anaplastic.
c) Spreads initially to iliac nodes via vaginal lymphatics.
d) Seldom involves lymph nodes at the time of presentation.
e) Is equally amenable to treatment by surgery and radiotherapy.
275- Treatment of cancer of the vulva includes all the following EXCEPT:
a) Radical excision of the vulva
b) Dissection of femoral triangle.
c) Unilateral or bilateral inguinal lymphadenectomy.
d) Pre-operative radiation.
e) Methotrexate.

276- Microinvasion of carcinoma of the cervix involves a depth below the base of the
epithelium of no more than:
a) 1 mm.
b) 2 mm.
c) 3 mm.
d) 4 mm.
e) 5 mm.

277- Invasive cancer of the cervix:


a) Is the commonest malignant tumor in women in the world.
b) Occurs most commonly in women under the age of 40 years.
c) Is usually of squamous type.
d) Occurs less commonly in smokers
e) Is uncommon in developing countries.

278- All the following about microinvasive cancer of the cervix are correct EXCEPT:
a) It consists only of those showing early stromal invasion.
b) It may invade to a depth of 5 mm.
c) It should have a maximum width of 7 mm.
d) It may invade lymphatic channels.
e) It may metastasize
279- Elevated α fetoprotein is used for detection of:
a) Cancer vulva.
b) Cancer cervix.
c) Endodermal sinus tumor.
d) Serous cystadenocarcinoma.
e) Mucinous cystadenocarcinoma.
280- The commonest vulval malignancy is:
a) Adenocarcinoma.
b) Squamous cell carcinoma.
c) Basal cell carcinoma.
d) Melanosarcoma.
e) Leiomyosarcoma.

281- Radiotherapy is indicated in endometrial carcinoma in the following situations


EXCEPT:
a) Preoperative RT is commonly performed to be followed by TAH + BSO.
b) Postoperative adjuvant therapy.
c) As primary treatment for the medically impossible.
d) To specifically treat pelvic side wall involvement.
e) Vaginal recurrence after hysterectomy.

282- In women with postmenopausal bleeding, it is best to perform endometrial


sampling if the endometrium on transvaginal ultrasound is thicker than:
a) 1 mm.
b) 2 mm.
c) 4 mm.
d) 6 mm.
e) 10 mm.

283- The definite diagnosis of a Leiomyosarcoma requires how many mitotic cells per
HPF?
a) 1.
b) 4.
c) 10.
d) 16.
e) 20.

284- The following are types of epithelial ovarian tumors EXCEPT:


a) Endometrioid adenocarcinoma.
b) Transitional cell tumor.
c) Brenner's tumor.
d) Thecoma.
e) Mucinous cystadenocarcinoma.

285- The most common primary site for atypical secondaries metastasizing to the ovary
is:
a) Stomach.
b) Lung. .
c) Colon.
d) Breast.
e) Esophagus.

286- As regards the spread of epithelial ovarian cancers, all the following are correct
EXCEPT:
a) Via the blood stream occurs early in the disease.
b) To para-aortic lymph nodes puts the case at stage III.
c) To the underside of the diaphragm is common.
d) Around the peritoneal cavity has usually occurred by the time of diagnosis.
e) To the omentum frequently occurs.
287- Important prognostic factors concerning ovarian epithelial carcinoma include the
following EXCEPT:
a) Volume of the tumor.
b) Affection of ovarian stroma
c) Extent of the tumor.
d) Histological differentiation of the tumor.
e) Presence of ascites.
288- The ovarian tumor that is best responds to radiotherapy is:
a) Serous cystadenocarcinoma
b) Mucinous cystadenocarcinoma
c) Transitional adenocarcinoma
d) Dysgerminoma
e) Granulosa cell tumor
289- The following factor are associated with endometrial cancer EXCEPT:
a) Obesity.
b) Smoking.
c) Polycystic ovarian disease and amenorrhea.
d) Heredity.
e) Tamoxifen use.
290- The most active chemotherapeutic drug in endometrial cancer is:
a) Paclitaxel.
b) Cisplatin.
c) Carboplatin.
d) Doxorubicin.
e) Cyclophosphamide.
291- The following hormonal treatment may be used in treating endometrial cancer
EXCEPT:
a) Provera (medroxy progesterone acetate).
b) The patients are younger.
c) LHRH analogues.
d) Tamoxifen.
e) Diethyl stilbestrol (DES).

292- Patient outcome is better if the endometrial cancer cells are:


a) Diploid.
b) Aneuploid.
c) Triploid.
d) Tetraploid.
e) Trisomy.
293- Postmenopausal bleeding is a common presentation of the following EXCEPT:
a) Cervical ectropion.
b) Carcinoma of the endometrium.
c) Atrophic vaginitis.
d) Carcinoma of the cervix.
e) Sarcoma of the uterus

294- The common presentations of carcinoma of the cervix include the following
EXCEPT:
a) An asymptomatic abnormal smear.
b) Pelvic pain.
c) Vaginal discharge.
d) Postcoital bleeding.
e) Intermenstrual bleeding.

295- All the following about CA 125 are correct EXCEPT:


a) Is a glycoprotein.
b) Is more commonly elevated in mucinous ovarian tumors than serous.
c) Is useful in screening for epithelial ovarian cancer.
d) Is a useful diagnostic marker in postmenopausal women with a pelvic mass.
e) Is accurate in predicting the presence of disease at second look laparotomy.

296- The following factors increase the risk of a woman developing endometrial cancer
EXCEPT;
a) Obesity.
b) Late menopause.
c) Combined (estrogen and progestin) hormone replacement therapy.
d) Diabetes mellitus.
e) A history of polycystic ovary disease.

297- Intra-peritoneal metastasis of a primary carcinoma of the ovary extending to the


surface of the liver with positive retro-peritoneal lymph nodes is consistent with:
a) Stage IF
b) Stage IIB.
c) Stage IIC.
d) Stage III.
e) Stage IV.

298- Which of the following statements about ovarian tumors in premenarcheal girls is
true?
a) The malignancy rate is low.
b) The majority are of celomic epithelial origin.
c) Ovarian neoplasia of any type are relatively common.
d) Most ovarian enlargement in the newborn are functional cysts.
e) Germ tumors are a frequent cause of precocious puberty.
299- The following are associated with a reduced prevalence of epithelial ovarian cancer
EXCEPT:
a) Unilateral oophorectomy.
b) Female sterilization.
c) IUCD usage.
d) Use of combined oral contraceptive pills
e) Previous hysterectomy.

300- The following factors make a diagnose of malignancy more likely in a woman with
an ovarian tumor EXCEPT:
a) Family history of ovarian cancer.
b) Previous use of the contraceptive pills.
c) Nulliparity.
d) Bilaterality of the tumor
e) Previous use of ovulation inducing drugs
301- Endometrial cancer with vaginal metastasis is staged:
a) IIb.
b) IIIa.
c) IIIb.
d) IIIc.
e) IVa.

302- Carcinoma in situ of the cervix precedes invasive for an average period of:
a) 1 year.
b) 2 years.
c) 5 years.
d) 10 years.
e) 15 years.

303- The following ovarian tumors are always malignant:


a) Myxoma peritonii.
b) Endodermal sinus tumor.
c) Solid teratoma.
d) Granulosa cell tumors.
e) Brenner tumors.

304- Risk factor for cancer ovary is:


a) Family history of cancer colon.
b) Use of progesterone only injectable contraception
c) Use of oral contraceptive pills.
d) Multiparity.
e) Serous ovarian cyst.

305- The following statements concerning endometrial cancer are correct EXCEPT:
a) Involvement of the cervix occurs in stage II disease.
b) Invasion of the myometrium to a depth of more than 50% puts the case into stage II.
c) Initial treatment of stage I disease is by total abdominal hysterectomy and bilateral
salpingo-oophorectomy.
d) Postoperative radiotherapy is rarely required in stage IA disease.
e) Five-year survival is 90% in stage I.

306- The following criteria are in favor of diagnosis of ovarian cancer during
laparotomy EXCEPT:
a) Huge ovarian cyst.
b) Extra cystic papillae.
c) Areas of hemorrhage and necrosis.
d) Heterogeneous consistency.
e) Limited mobility

307- Which of the following germ cell tumors is associated with ambiguous genitalia?
a) Choriocarcinoma.
b) Gonadoblastoma.
c) Dermoid cyst.
d) Carcinoid tumor.
e) Endodermal sinus tumor.
308- True statements about mucinous carcinoma of the ovary include the following
EXCEPT:
a) It usually is diagnosed at a less advanced stage than serous carcinoma.
b) It tends to be unilateral.
c) It tends to be well differentiated.
d) It may be developed in 30-50% of benign forms.
e) Affected women have a 50 percent chance of surviving 5 years.

309- Adjuvant chemotherapy may be indicated in all the following EXCEPT:


a) Stage I A cancer ovary.
b) Stage I c cancer ovary.
c) Cancer ovary stage II.
d) Bulky cervical carcinoma.
e) Cancer vulva.

310- The following statements regarding endometrial carcinoma are true EXCEPT:
a) Age is a significant prognostic factor.
b) Transtubal dissemination explains metastatic intra-abdominal spread.
c) Spillage of cancer cells at the time of surgery is the cause of vaginal vault
metastases.
d) One third of all cases of relapse will occur within 12 months of diagnosis.
e) Ten percent of cases of relapse occur at more than five years from surgery.

311- Women arc at high risk for endometrial carcinoma if they have one or more of the
following characteristics EXCEPT:
a) Hypertension.
b) Diabetes.
c) Smoking
d) Obesity.
e) Familial history of endometrial carcinoma.

312- Risk factor for cancer vulva is:


a) HIV infection.
b) Vulval dystrophy.
c) Lichen sclerosis.
d) Herpes simplex.
e) Bartholin abscess.

Directions Questions 313 through 347: Each set of matching questions in this section
consists of a list of 4 to 8 lettered options followed by several numbered items. For each
item, select the ONE best lettered option that is most closely associated with it. Each
lettered heading may be selected once, more than once, or not at all.

Questions 313-317
Match the mean age of occurrence and the genital malignancy
a) 4-5 years
b) 10-20 years
c) 45-50 years
d) 57-65 years
313- Endometrial cancer
314- Sarcoma botryoid
315- Cervical cancer
316- Vulval cancer
317- Dysgerminoma

Questions 318-323
Match the main type of spread of the following malignancies and its corresponding
tumor
a) Tumor is locally malignant
b) Lymphatic
c) Hematogenous
d) Transcelomic

318- Serous cystadenocarcinoma of the ovary


319- Dysgerminoma of the ovary
320- Malignant melanoma of the vulva
321- Basal cell carcinoma of the vulva
322- Squamous cell carcinoma of the vulva
323- Squamous cell carcinoma of the cervix

Questions 324-332
Match the approximate malignant potential of the following benign condition
over a decade with the condition of concern
a) 0%
b) 0.5%
c) 1% - 1.5%
d) 3%
e) 5%
f) 8%
g) 30%

324- Ovarian fibroma


325- Ovarian mucinous cystadenoma
326- Ovarian dermoid cyst
327- Simple endometrial hyperplasia without atypia
328- Complex endometrial hyperplasia without atypia
329- Simple endometrial hyperplasia with atypia
330- Complex endometrial hyperplasia with atypia
331- Uterine leiomyoma
332- Cervical mucous polyp

Questions 333-337
Match the tumor marker or byproduct with the appropriate tumor
a) Human chorionic gonadotrophin
b) Alpha-fetoprotein
c) CA 125
d) Estrogen
e) Progesterone
f) Testosterone
g) Non of the above
333- Dysgerminoma
334- Choriocarcinoma
335- Theca cell tumor
336- Endodermal sinus tumor
337- Serous cystadenocarcinoma

Questions 338-342
Match the chemotherapeutic agent with its characteristic side effect
a) Pulmonary fibrosis
b) Cardiotoxicity
c) Peripheral neuropathy
d) Renal failure
e) Hemorrhagic cystitis

338- Adriamycin
339- Vincristine
340- Cyclophosphamide
341- Cisplatinum
342- Bleomycin
Questions 343-347
Match the findings with the least stage of the tumor
a) Stage I A
b) Stage TB1
c) Stage I B2
d) Stage IIA
e) Stage IIB
f) Stage HI
g) Stage IVA h) Stage IVB

343- Tumor size of 5 cm which is confined to the cervix


344- Tumor of size 3 cm which extends to the pelvic wall
345- Tumor size of 4 cm with multiple hepatic secondaries
346- Tumor size of 4 cm with bilateral hydronephrosis
347- Tumor size of 2 cm with affection of 2 cm of the vaginal vault

Directions: Each of the following numbered items or incomplete statements in this


section is followed by numbered items that in certain combination leads to the BEST
lettered combined answer or/and completion. According to the following directions,
choose the BEST lettered answer:
If
1,2,3 are correct, choose A
1,3 are correct, choose B
2,4 are correct, choose C
1,2,3,4 are correct, choose D
All are correct, choose E

348- Primary carcinoma of the vagina:


1) More commonly occurs in the upper vagina than in the lower.
2) Is associated with cervical cancer.
3) Is usually treated with radiotherapy.
4) Usually presents as pelvic pain.
5) Is usually an adenocarcinoma.

349- All the following about invasive cancer of the vulva are correct:
1) Is usually squamous in type.
2) Carries a poor prognosis.
3) Is associated with other genital malignancies.
4) Is best treated by local vulvectomy.
5) Is painful.

350- Incidence of all of the following is increased with tamoxifen use:


1) Endometrial polyp.
2) Twin pregnancy
3) Endometrial adenocarcinoma.
4) Cervical dysplasia.
5) Uterine fibroids.

351- Tumor marker are reliable for following of:


1) Serous cystadenocarcinoma of ovary.
2) Granulosa cell tumor.
3) Germ cell ovarian tumor (endodermal sinus tumor).
4) Cancer cervix.
5) Cancer vulva.

352- Carcinoma of the ovary:


1) Cancers of epithelial origin comprise 90% of total primary ovarian malignancies.
2) Borderline tumors may metastasize but do not invade adjacent tissue.
3) Transcelomic metastases are a common finding at laparotomy.
4) HRT may reasonably be prescribed after surgery of ovarian cancer.
5) Is predisposed to by prolonged use of the oral contraceptive pill.

353- Fibromas of the ovary:


1) Commonly present as a large abdominal mass.
2) Frequently become malignant.
3) May be associated with a hydrothorax.
4) Is usually cystic
5) Often contain teeth.

354- Tn a women presenting with postmenopausal bleeding:


1) Hysteroscopy is a useful diagnostic tool.
2) The most common cause of bleeding is endometrial cancer.
3) Sonographic measurement of endometrial thickness is useful.
4) Benign causes of bleeding are rare.
5) Dilatation and curettage is almost always accurate.

355- Cancer ovary commonly presents by:


1) Abdominal swelling.
2) Menstrual disturbance.
3) Gastrointestinal upset.
4) Urinary retention.
5) Change, in voice.
356- Cancer ovary staging depends upon:
1) Surgical exploration.
2) U/S examination.
3) Surgical sampling of peritoneal fluid & lymph node sampling and omental biopsy.
4) MRI examination.
5) Clinical examination only.

357- Endometrial carcinoma:


1) 5% occur in women below the age of 40 years.
2) Obesity is a recognized risk factor.
3) 80% of women present with postmenopausal bleeding.
4) Screening is advisable for all women.
5) Treatment is essentially by radiotherapy.

358- The following tumors of the ovary are commonly hormone secreting:
1) Serous cystadenomas.
2) Granulosa cell tumors.
3) Transitional cell tumors.
4) Leydig cell tumor.
5) Mature cystic teratomas.

359- In endometrial carcinoma, progestogen therapy is useful in the following


circumstances:
1) Well-differentiated endometrial carcinoma lesions.
2) Steroid receptor positive endometrial carcinoma patients.
3) As an adjuvant to surgery.
4) Recurrent endometrial carcinoma.
5) In the treatment of pulmonary metastases.

360- Differential diagnosis of vulval cancer include:


1) Granuloma inguinale.
2) Syphilis.
3) Herpes simplex type II infection.
4) Rodent ulcer.
5) Condyloma accuminata.

361- In stage II carcinoma of the cervix, the following are correct:


1) The upper third of the vagina may be involved.
2) The tumor is fixed to the lateral pelvic wall.
3) There may be extension into the body of the uterus.
4) A five-year survival rate of 80% can be expected.
5) The growth is confined to the cervix.

362- The following about cancer cervix are correct:


1) Adenocarcinoma occurs in about 10% of cases.
2) Is the most common cancer among women.
3) Is more common in smokers.
4) Does not spread by direct infiltration.
5) Is more common in nulliparous women.

363- Symptoms of cervical cancer may include:


1) Perimenopausal bleeding.
2) Uremia.
3) Postmenopausal bleeding.
4) Incontinence of urine.
5) Severe pain.

364- Dysgerminomas:
1) Are commoner in younger than older women.
2) Frequently present with abdominal pain.
3) Are common in patients with testicular feminization syndrome.
4) Are treated initially by radiotherapy.
5) Are bilateral in 50% of cases.

365- Dysgerminoma of ovary:


1) Is common below the age of 30.
2) Recurs late, over ten years.
3) Is radio-sensitive.
4) May produce alpha-fetoprotein (AFP).
5) Is associated with gonadal dysgenesis.

366- Chemotherapeutic agents used to treat epithelial ovarian cancers:


1) Are more effective after cytoreductive surgery.
2) Are best given continually for six months.
3) Should contain platinum for best effect.
4) Usually results in a cure.
5) Should be used for all stages of the disease.

367- In ovarian cancer:


1) Advanced disease is seen in more than 50% of cases.
2) For all stages combined, the five-year survival is less than 30%.
3) Survival is related to the residual tumor present postoperatively.
4) Retroperitoneal nodal metastases are rare.
5) Autosomal dominant inheritance may occur.

368- FIGO staging of cancer cervix:


1) Stage IIIa denotes involvement of the lower 1/3 of the vagina.
2) 20% include of lymph nodes involvement in stage IB.
3) Cystoscopy is essential for proper staging.
4) Rectal examination is useful in the staging system.
5) Is a surgical staging system.

369- In assessing the histopathological type of cervical intraepithelial neoplasia (CIN)


present, the following factors are taken into account:
1) Numbers of mitotic figures.
2) The nuclear-cytoplasmic ratio.
3) Epithelial differentiation.
4) Crypt involvement.
5) The presence of human papilloma virus (HPV).

370- When using exfoliative cytology to diagnose cervical neoplasia:


1) The aim is to sample the surface cells of the cervical transformation zone.
2) The sampling device must cover 360° of the cervix.
3) Fixation must take place immediately.
4) Aspiration of cells from the posterior fornix pool is less useful than using an Ayre
spatula to scrape the cervical surface.
5) The presence of endometrial cancer will be detected in 70% of cases.
Directions: In the following questions, each item has 5 lettered options each of which
could be a true or a false statement. Tick each of these options according to whether
they are false or true

371- As regard vulval intraepithelial neoplasia:


a) Paget's disease is considered a variant of VIN.
b) VIN-I can be visible macroscopically.
c) Diagnosis of VIN is best archived by cytology and colposcopy.
d) VIN can be treated by CO2 laser to a depth of 3 mm.
e) VIN can be treated by wide local excision to a depth of 5 mm.

372- In diagnosis of CIN:


a) Pap smear is best method for diagnosis.
b) HPV DNA detection can be used.
c) Low grade squamous intra epithelial lesions include CIN I, II.
d) Abnormal Pap smear is an indication for cervical biopsy.
e) Abnormal Pap smear with colposcopy is an indication for colposcopy directed
biopsy.

373- As regards the prognosis of cervical cancer:


a) Recurrence occurs in 35% of cases.
b) The majority of recurrence occurs after 3 years of treatment.
c) The 5 years- survival for stage II is 50%.
d) Surgical management of local recurrence following radical surgery is disappointing.
e) In stages IB & IIA there is little difference between results of surgery &
radiotherapy.

374- The following statements apply to carcinoma of the cervix:


a) Adenosquamous and tumors of mixed histological type all arise from transition zone
epithelium.
b) Ninety per cent are pure squamous cell tumors.
c) Mixed adenosquamous tumors are associated with poorer survival rates than pure
adenocarcinomas.
d) The presence of vascular space permeation is a prognostic indicator independent of
lymph node status.
e) The risk of lymph node metastases in women with microinvasive disease is 8.5%.

375- A patient with stage Ib carcinoma of the cervix undergoes a radical hysterectomy
and pelvic lymphadenectomy:
a) Prophylactic heparin is mandatory.
b) Irradiation is an alternative comparable therapeutic modality
c) Ureteric fistulae are usually due to intra-operative surgical trauma.
d) Significant long-term bladder dysfunction is common.
e) Pelvic lymphocyst formation is characteristically a late complication.
376- A 28-year-old woman undergoes a radical hysterectomy including pelvic
lymphadenectomy for a 3 cm diameter stage Ib squamous cell carcinoma of the
cervix:
a) Her prognosis will be better after surgery than following radiotherapy.
b) Pre-operative intracavitary cesium reduces recurrence.
c) Oophorectomy is unnecessary.
d) Post-operative radiotherapy would improve the prognosis if the pelvic nodes
contained tumor.
e) Para-aortic lymphadenectomy should be employed as a routine at the time of radical
hysterectomy.

377- Endometrial cancer:


a) Constitutes 25-30% of all gynecological malignancies.
b) The commonest type is adenosquamous.
c) Pelvic nodes are involved in 5% of poorly differentiated cases.
d) Adenosquamous carcinoma has a better prognosis than adenocarcinoma.
e) Has a greater tendency to metastasize if it involves the lower uterus than the fundus.

378- Concerning second-look laparotomy in epithelial ovarian cancer:


a) This should be restricted to research protocols only.
b) This has no effect on long-term prognosis.
c) This should be considered in all patients with advanced epithelial ovarian cancer.
d) This applies especially to patients who have not responded to chemotherapy.
e) Up to 50 per cent of patients with a negative second-look laparotomy will have a
recurrence.
FAMILY PLANNING
Directions: Each of the numbered items or incomplete statements in this section is
followed by answers or by completions of the statement. Select the ONE lettered answer
or completion that is BEST in each case.

379- Which of the following listings correctly ranks contraceptive methods in terms of
decreasing effectiveness:
a) Oral contraceptives, diaphragm, IUD, spermicides, rhythm.
b) IUD, oral contraceptives, diaphragm, spermicides, rhythm.
c) Rhythm, oral contraceptives, IUD, diaphragm, spermicides.
d) Oral contraceptives, IUD, spermicides, diaphragm, rhythm.
e) Oral contraceptives, IUD, diaphragm, spermicides, rhythm.

380- The use of combined oral contraceptives reduces the risk of:
a) Ectopic pregnancy.
b) Hepatic adenoma.
c) Salpingitis.
d) Ovarian cancer.
e) Endometrial cancer.

381- Regarding the progesterone only pill:


a) 60% of women using it will ovulate.
b) Ideally it should be taken just before bedtime.
c) The pearl index is higher in the older reproductive age group.
d) Is associated with a lower risk of an ectopic pregnancy than for a non-user.
e) Is at least as effective as the combined contraceptive pill.

382- AH these methods could be used during lactation EXCEPT:


a) Depot provera.
b) Mini pills.
c) Combined contraceptive pills.
d) I.U.D.
e) Male condom.

383- Mechanism of action of copper IUD includes:


a) Inhibition of ovulation.
b) Tubal block.
c) Mechanical inflammatory reaction of the endometrium.
d) Increased tubal motility.
e) Cervical mucus hostile to the sperms.

384- All the following are health benefits of combined oral contraceptive pills EXCEPT:
a) Endometrial carcinoma protection.
b) Protection against surface ovarian tumors.
c) Treatment of benign breast lesions.
d) Protection against cancer breast.
e) Decrease amount of menstrual flow.

385- The side effects of combined oral contraceptive pills include the following
EXCEPT:
a) Nausea.
b) Dizziness.
c) Vaginal discharge.
d) Menorrhagia.
e) Weight gain.

Directions Questions 386 through 394: Each set of matching questions in this section
consists of a list of 6 lettered options followed by several numbered items. For each
item, select the ONE best lettered option that is most closely associated with it. Each
lettered heading may be selected once, more than once, or not at all.

Questions 386-390
Match the following contraceptive method with the most likely associated condition:
a) Combined oral contraceptives
b) Progestin-only pill
c) Condoms
d) Cervical cap
e) Intrauterine device (IUD)
f) Coitus interruptus

386- Toxic shock syndrome


387- Cholelithiasis
388- Actinomyces Israeli
389- Highest risk of contraceptive failure
390- Dysmenorrhea

Questions 391-394
For each woman, select the LEAST appropriate contraceptive method
a) Combined oral contraceptives
b) Progestin-only pill
c) Levonorgestrel implant
d) Condoms
e) Diaphragm
f) Intrauterine device (IUD)

391- A 17-year-old woman with a history of ectopic pregnancy


392- A 25-year-old woman who is nursing
393- A 38-year-old woman who smokes 40 cigarettes daily
394- A 37-year-old woman with a large cystocele

Directions: Each of the following numbered items or incomplete statements in this


section is followed by numbered items that in certain combination leads to the BEST
lettered combined answer or/and completion. According to the following directions,
choose the BEST lettered answer:
If
1,2,3 are correct, choose A
1,3 are correct, choose B
2,4 are correct, choose C
1,2,3,4 are correct, choose D
All are correct, choose E

395- The absolute contraindications use of combined oral contraceptive pills:


1) D.V.T
2) Heavy smoking.
3) Sickle cell anemia.
4) Hypertension.
5) Diabetes mellitus.

396- The following methods protect against sexually transmitted diseases:


1) Male condom.
2) Diaphragm.
3) Female condom.
4) Combined contraceptive pills.
5) IUD.

397- The following are contraindications to using combined oral contraceptives:


1) Pulmonary embolus.
2) Porphyria.
3) Sickle-cell disease.
4) Previous GIN III.
5) Depression.

398- The following statements regarding progestogen only oral contraceptive


preparations are correct
1) They are a suitable method of contraception for breast feeding women.
2) Woman needs to take them at the same time each day.
3) They are more reliable as the patient's age increases.
4) They are contraindicated in smokers over the age of 40.
5) They are as reliable as the combined oral contraceptive pill in preventing pregnancy.

399- The following statements regarding progestin only injectables as contraceptives are
correct:
1) They have no effect on lactation and can be used by breastfeeding women.
2) The mechanism of their action is mainly by suppression of ovulation.
3) Return to fertility after discontinuation is usually delayed.
4) They can cause osteoporosis.
5) DMPA (Depo-medroxyprogesterone acetate) is taken every month.

400- The following statements regarding norplant are correct:


1) The main mechanism of action is hostility of the cervical mucus.
2) It contains estrogen.
3) It is composed of the Progestin: levonorgestrel.
4) It is irreversible contraceptive.
5) Side effects are rare.

Directions: In the following questions, each item has 5 lettered options each of which
could be a true or a false statement. Tick each of these options according to whether
they are false or true
401- Emergency contraception:
a) Is contraception used after intercourse but prior to implantation.
b) When used estrogens must be taken within 72 hours of intercourse.
c) When using estrogens, rarely they have side effects.
d) When using the IUCD, it must be inserted the "morning after".
e) Established contraindications to the oral contraceptive pill and the intrauterine
contraceptive device still apply.

402- The combined oral contraceptive pill:


a) Is safe for women to use till the menopause provided they are healthy non-smokers.
b) Containing third-generation progestogens are more androgenic than pills containing
norethisterone.
c) May be associated with a slightly increased risk of early onset breast cancer in
women who start to use it in their teens.
d) Is associated with a reduction of almost 50% in the risk of both endometrial and
ovarian cancer.
e) A high-dose (50 ug estrogen) preparation should be used by women taking the
anticonvulsant sodium valproate.

403- Which of the following statements with regard to the copper IUCD are correct?
a) Modern copper IUCDs are clinically effective and safe for at least five years.
b) It reduces the number of sperm reaching the fallopian tube and their capacity to
fertilize the egg.
c) The risk of pelvic infection (PID) is low (<2 cases PID/1000 woman years of use)
and does not increase with long-term use.
d) Is contraindicated in women with Wilson's disease.
e) Levonorgestrel releasing devices are associated with a highly significant reduction in
blood loss in women with menorrhagia.

404- In hormonal contraception:


a) Ethinyl Estradiol (EE) is the estrogen present in the combined oral contraceptive
pills (COCs).
b) Progestins are synthetic compounds that mimic the structure of natural progesterone.
c) Failure rate is about 1/HWY with COCs and 5/HWY with POPs.
d) Combined oral contraceptive pills reduced risk of anemia.
e) History of hepatic adenoma is a contraindication of COC pills.

405- Intrauterine devices:


a) As a mechanism of action, it neither affects ovulation nor steroidogenesis.
b) Failure rate is 5-10/HWY.
c) Perforation of the uterus is rare.
d) When intrauterine pregnancy occurs on top of IUCD spontaneous abortion risk of
50% if the IUCD is left and 25% if removed.
e) If a pregnancy occurs in IUCD user, the risk of it being ectopic is lower than in
women using other contraceptive methods.
OBST E T RICS
NORMAL PREGNANCY

Directions: Each of the numbered items or incomplete statements in this section is


followed by answers or by completions of the statement. Select the ONE lettered answer
or completion that is BEST in each case.

406- Fertilization:
a) Occurs in the uterine cavity.
b) If occurred by two spermatozoa, causes a trisomic conceptus.
c) Is associated with a surge of maternal luteinizing hormone.
d) Is associated with production of the first polar body.
e) Depends on hyaluronidase release by the sperm.

407- Which of the following is NOT characteristic of arachidonic acid?


a) The non-esterified form is a precursor of prostaglandin.
b) The esterified form is stored in the decidua vera.
c) It is found in high levels in the amniotic fluid.
d) It is liberated from the esterified form by phospholipase.
e) It combines with prostaglandin synthetase to produce prostaglandin.
408- Which of the following is NOT involved in the synthesis of prostaglandin?
a) Fetal membranes.
b) Phospholipase.
c) Esterified arachidonic acid.
d) Prostaglandin synthetase.
e) Progesterone.
409- Succenturiate placenta could be associated with:
a) Antepartum hemorrhage.
b) Postpartum hemorrhage.
c) Preterm delivery.
d) Postterm delivery.
e) Congenital fetal malformations.
410- Which of the following hormones is NOT a product of placental synthesis or
production:
a) Human chorionic gonadotropin (hCG).
b) Human placental lactogen (hPL).
c) Dopamine
d) Progesterone.
e) Estriol.
411 - Which is NOT a characteristic of human placental lactogen, the growth hormone
of pregnancy:
a) It elevates free fatty acids.
b) It elevates plasma insulin levels.
c) It induces lipolysis.
d) It inhibits gluconeogenesis in the mother.
e) It stimulates glucose uptake in the mother.
412- Which of the following is NOT a characteristic of progesterone:
a) It is an intermediary product in steroid metabolism.
b) It contains 21 carbon atoms.
c) Its main source during early pregnancy is the corpus luteum of pregnancy.
d) It is a precursor of testosterone.
e) Its ovarian source is important after the first 9 weeks of pregnancy.

413- In the normal fetus and newborn, the following statements are correct EXCEPT:
a) Prostaglandins delay closure of the ductus arteriosus after birth
b) Glucose reaches his circulation by facilitated diffusion.
c) The central nervous system is fully mature at birth.
d) The ductus venosus conveys oxygenated blood before birth.
e) There is high pulmonary vascular resistance before birth.

414- The following vessels contain oxygenated blood in the fetus EXCEPT:
a) Umbilical artery.
b) Ductus venosus.
c) The inferior vena cava as it enters the right atrium.
d) Carotid artery.
e) Umbilical vein.

415- In early development:


a) The amnion is a double layer of fetal mesodermal origin.
b) The decidua capsularis is a component of the chorion.
c) Chorionic villi are the functional hormonal units.
d) Human chorionic gonadotrophin is produced mainly by cytotrophoblast.
e) Relaxin is produced by the chorion laeve.

416- The functional unit of the placenta is:


a) The decidua
b) The chorionic villus
c) The cotyledon
d) The cytotrophoblast
e) The chorion laeve

417- The following statements regarding human chorionic gonadotrophin (hCG) are
correct EXCEPT:
a) Is a glycoprotein.
b) Has an a subunit similar to FSH.
c) Reaches a peak level at about 20 weeks' gestation.
d) Is thought to stimulate fetal testosterone secretion.
e) Is produced by syncytiotrophoblast.

418- Softening of the cervical isthmus that occurs early in gestation is called:
a) Hegar's sign.
b) Chadwick's sign.
c) Braxton-Hick's contraction.
d) Palmer's sign.
e) Cullen's sign.

419- A pregnant uterus that is larger than the period of amenorrhea could be due to:
a) Generalized edema.
b) Obesity.
c) Intrauterine growth retardation.
d) Breech presentation.
e) Polyhydramnios.

420- Which of the following signs or symptoms are NOT present in a 12-week
pregnancy:
a) Chadwick's sign.
b) Quickening.
c) Ultrasonographic fetal heart action.
d) Amenorrhea.
e) Hegar's sign.

421- A pregnant vegetarian is likely to be deficient in which of the following substances?


a) Calcium.
b) Folic acid.
c) Iron.
d) Protein.
e) Vitamin B12.

422- Which of the following is NOT a basis in Naegele's rule for estimating a woman's
due date?
a) Regular monthly menstrual cycles.
b) A pregnancy of 280 days.
c) Ovulation about day 14.
d) Cycle regulation with birth control pills before conception.
e) Conception at midcycle.

423- Which of the following is NOT a usual screening test in an early, uncomplicated
pregnancy?
a) Repeat human chorionic gonadotropin (hCG) levels.
b) Hemoglobin.
c) Serology.
d) Cervical cytology.
e) Blood type and Rh factor.

424- Which of the following dietary instructions NOT appropriate for a pregnant
woman?
a) Restrict salt intake.
b) Take 1200 mg calcium daily.
c) Take 800 mg folic acid daily.
d) Take supplemental iron.
e) Gain at least 15 Ibs during the pregnancy.

425- Which of the following complications does NOT occur in the second trimester of
pregnancy:
a) Premature labor.
b) Cervical incompetence.
c) Premature rupture of membranes.
d) Abruptio placentae.
e) Round ligament pain.
426- Which of the following is NOT a characteristic of a 28-week pregnancy:
a) Viability
b) A fetal weight of 1000 g.
c) Lecithin to sphingomyelin (L/S) ratio of less than 2:1
d) The absence of type II fetal lung alveoli cells
e) The presence of phosphatidyl glycerol.
427- The INCORRECT statement about fetal scalp PH measurement:
a) It is a good indicator of chronic fetal hypoxia.
b) It is measured using an invasive test.
c) It can be repeated every hour if abnormal.
d) Normal value of fetal PH should be >7.25.
e) Maternal pH may affect the fetal pH

428- The correct statement about biophysical profile is:


a) It is made up of four components.
b) A score of 6 is satisfactory.
c) Perinatal mortality rises with falling biophysical profile score.
d) Amniotic fluid volume is an important sign of acute asphyxia.
e) It consists of Doppler flow readings of the umbilical cord.

429- Management of intrapartum acute fetal distress includes the following EXCEPT:
a) Oxygen administration.
b) Change of maternal position.
c) Intraamniotic oxygen supplement
d) Correction of maternal hypotension.
e) C.S if the cervix is not fully dilated.

430- The following statements regarding obstetric ultrasound are correct EXCEPT:
a) It can be used with amniocentesis.
b) It carries a major risk to the fetus.
c) It can diagnose placental grading.
d) It is a useful tool in the assessment of amniotic fluid volume.
e) It could estimate the approximate intrauterine fetal weight

431- Ultrasound can be helpful in the diagnosis of all the following EXCEPT:
a) Incompetent cervix.
b) Fetal cardiac anomalies.
c) Type of twins.
d) Site of implantation
e) Placental location

432- Amniocentesis is useful in the following situations EXCEPT:


a) Suspected open neural defect.
b) Management of un-sensitized Rh-negative pregnant women.
c) Tests for fetal lung maturity.
d) Suspected chorioamnionitis.
e) Examine fetal cells for the chromosomal pattern.
433- The count-to-ten chart to record fetal movements:
a) Is an accurate method to assess the fetal wellbeing.
b) It is the time needed to appreciate ten fetal movements.
c) It is the number of fetal movements in ten minutes.
d) If the result is unsatisfactory, immediate delivery of the fetus is indicated.
e) If the result is satisfactory, the fetus is definitely in safe condition.

434- Fetal lung maturation is ensured by the presence of:


a) Lecithin.
b) Prostaglandin.
c) Sphingomyelin.
d) Phosphatidyl glycerol.
e) Cortisol.

435- In the fetal circulation, the INCORRECT statement is:


a) There are two umbilical veins & one umbilical artery.
b) The ductus venosus connects the umbilical vein with IVC.
c) The blood is shifted from the right atrium to the left atrium via the foramen oval.
d) Prostaglandins maintain patency of ductus arteriosus.
e) The inferior vena cava contains both oxygenated & deoxygenated blood.

Directions Questions 436 through 455: Each set of matching questions in this section
consists of a list of 4 to 8 lettered options followed by several numbered items. For each
item, select the ONE best lettered option that is most closely associated with it. Each
lettered heading may be selected once, more than once, or not at all.

Questions 436-440
Match the drug used in pregnancy with its presumptive effect on the fetus
a) Ototoxicity
b) Bone affection
c) Kernicterus
d) Cardiotoxicity
e) Grey baby syndrome
f) Neural tube defect

436- Chloramphenicol
437- Sulphonamides
438- Carbamazepine
439- Tetracycline
440- Gentamicin

Questions 441-444
Match the fetal structure with the adult correspondence
a) Ligamentum venosum
b) Ligamentum teres
c) Ligamentum arteriosum
d) Median umbilical ligament
e) Lateral umbilical ligament
441- Umbilical artery
442- Ductus venosus
443- Ductus arteriosus
444- Urachus
Questions 445-450
Match the substance with its method of placental transfer
a) Active transport
b) Facilitated diffusion
c) Simple diffusion
d) pinocytosis
e) Not transferred

445- Glucose
446- Non fractionated heparin
447- Fractionated heparin
448- Amino-acids
449- Oxygen
450- Immunoglobulins

Questions 451-455
Match the physiologic condition in the non pregnant state below with its
change during pregnancy above
a) Increases
b) Decreases
c) Remains constant

451- Intestinal peristalsis


452- Serum cortisol level
453- Serum thyroxine level
454- Respiratory rate
455- Cardiac output

Directions: Each of the following numbered items or incomplete statements in this


section is followed by numbered items that in certain combination leads to the BEST
lettered combined answer or/and completion. According to the following directions,
choose the BEST lettered answer:
If
1,2,3 are correct, choose A
1,3 are correct, choose B
2,4 are correct, choose C
1,2,3,4 are correct, choose D
All are correct, choose E

456- The following statements regarding fertilization are correct:


1 - It occurs in the ampullary region of the fallopian tube.
2- The sperm penetrates the ovum by the action of prostaglandins.
3- After penetrating the ovum, the head of the sperm swells to form the male
pronucleus.
4- The fertilized ovum starts to divide by meiotic division to form the zygote.
5- Multiple sperms succeed in penetrating the ovum.
457- Concerning the decidua, the following statements are correct:
1 - It is a specialized endometrium of pregnancy.
2- It is the site of implantation of the blastocyst.
3- It is a protective layer against the invasive power of the trophoblast.
4- It has a relation with trophoblast that follows the laws of transplantation
immunology.
5- The decidua parietalis shares in the formation of the placenta.

458- In human placentation, the following structures lie between maternal and fetal
blood:
1 - Trophoblast.
2- Capillary endothelium.
3- Mesenchyme.
4- Decidua.
5- Chorion laeve.

459- Regarding the Placenta, the following statements are correct:


1 - It is formed by the decidua vera & chorion frondosum.
2- Placental hormones are secreted by the Syncytium.
3- Placental transfer occurs mainly by simple diffusion.
4- Hydatidiform mole is a benign placental tumor.
5- It is usually large in intrauterine growth retardation.

460- Regarding the normal umbilical cord, the following statements are correct:
1 - It contains two arteries.
2- It is covered by amnion and chorion.
3- It contains Wharton's jelly.
4- It usually pulsates at a rate equal to the maternal pulse rate.
5- It contains two veins.

461- The following statements regarding estrogens in pregnancy, are correct:


1- E3 (estriol) is the main estrogen during pregnancy.
2- They are decreased in cases of anencephaly.
3- They are secreted by the syncytium.
4- They stimulate the alveolar development in breasts.
5- They are of maternal source only.
462- The following statements regarding human chorionic gonadotrophins (hCG) are
correct:
1 - It is secreted by the syncytium.
2- It starts to be measurable in maternal blood 10 days post conception.
3- It is luteotropic.
4- Its maximum serum level is at 10 weeks of gestation.
5- Its secretion is controlled by the placenta, ovary and pituitary gland.

463- The following statements regarding the placenta the following statements are
correct:
1 - It is formed of maternal and fetal parts.
2- The placental barrier get thinned by 5-6 months of pregnancy.
3- Placenta succenturiata may be a cause of post partum hemorrhage.
4- Its maternal surface is covered by an amniotic membrane.
5- Each cotyledon is composed of a few numbers of chorionic villi.
464- The following statements regarding the amniotic fluid are correct:
1 - It has a fetal contribution as well as maternal one.
2- It has functions during pregnancy as well as during labor.
3- Its marked diminution may lead to skeletal abnormalities in the fetus.
4- It is increased in anencephaly.
5- Its PH is acidic.

465- As regards the amniotic fluid, the following statements are correct:
1 - The water constitutes 99% of its composition.
2- It reaches its maximum volume at term.
3- It keeps the fetal temperature constant.
4- In the first trimester, it is mainly of fetal origin.
5- It is a stagnant fluid.

466- As regards a feto protein the following statements are correct:


1- It is reduced concentration in the maternal serum in Down's syndrome.
2- It is produced by the yolk sac.
3- It constitutes apart of the triple test
4- Reaches its highest concentration in the maternal serum at about 16 weeks
gestation.
5- Has a similar concentration in fetal serum and amniotic fluid.

467- The following statements regarding human placental lactogen (HPL) are correct:
1 - It elevates free fatty acids.
2- It inhibits gluconeogenesis.
3- It induces lipolysis.
4- It decreases plasma insulin levels.
5- It stimulates glucose uptake in mother.

468- The following statements regarding the umbilical cord are correct:
1 - Contains two arteries and one vein.
2- Abnormal long cord can lead to cord prolapse.
3- Contains Wharton's Jelly.
4- It is covered with amniotic membrane.
5- Its umbilical vein carries oxygenated blood from placenta to the fetus.
469- Warning symptoms that are given to a pregnant lady include:
1- Bleeding per vagina.
2- Sudden loss of fluid per vagina.
3- Abdominal pain.
4- Leg cramps.
5- Excessive salivation (ptyalism).
470- The following laboratory investigations are component of the routine antenatal
care
1 - Rh typing.
2- Hemoglobin.
3- Urine analysis for sugar and protein.
4- HCG
5- Urine culture.
471- Cardiac output during normal pregnancy:
1 - Increase in the first trimester.
2- Varies with physiological changes in heart rate.
3- Varies with stroke volume when the heart rate is constant.
4- Is reflexly reduced in a hot environment.
5- Is greater from the left ventricle than from the right.
472- The following statements regarding blood composition in normal pregnancy are
correct:
1- The packed cell volume falls.
2- There is a rise in the iron-binding capacity.
3- The blood cholesterol rises.
4- The total red cell mass falls by about 20%
5- The protein bound iodine level falls.

473- The following statements regarding non-stress test (NST) are correct:
1- Used with the aid of internal fetal Doppler.
2- Long term variability of the fetal heart beats is a good sign.
3- Late deceleration is a reassuring sign.
4- Short term variability of the fetal heart beats is a good sign.
5- Normal baseline fetal heart rate at term is 100.
474- The following statements regarding Manning biophysical profile of the fetus are
correct:
1 - Score ten is the maximum value.
2- Score zero is given for dead fetus.
3- Score 6 is non-reassuring
4- It includes four parameters.
5- It can be performed starting at 20 weeks gestational age.
475- Circulatory changes in a healthy woman during normal pregnancy include:
1 - A uterine blood flow at term of the order of 500 ml/minute.
2- A continuous increase in stroke volume.
3- A maximum increase in resting cardiac output is from 30% to 60%.
4- A rise in cardiac output only during the second and third trimesters.
5- The peripheral blood flow is reduced.

476- In late pregnancy, an increase occurs in the blood concentration of:


1 - Fibrinogen.
2- Transferrin.
3- Cholesterol
4- Albumin.
5- Sodium.

477- Recognized features of Turner's syndrome include:


1 - Coarctation of the aorta.
2- Hypogonadotrophic hypogonadism.
3- Increased carrying angle.
4- Increased incidence with advanced maternal age.
5- Anosmia.

478- Recognized features of Turner's syndrome include:


1 - Coarctation of the aorta.
2- Elevated serum gonadotrophin levels.
3- Neonatal peripheral edema.
4- Chromatin positive buccal cells.
5- Anosmia.

479- Progesterone hormone:


1- Its main source during early pregnancy is the corpus luteum of pregnancy.
2- It is the precursor of testosterone.
3- It is an intermediate product in steroid metabolism.
4- It is an index of the maternal-fetal placental unit.
5- Its ovarian source is important after the first 9 weeks of pregnancy.

480- With regard to normal pregnancy, the following statements are correct:
1- In the middle trimester glomerular filtration rate is increased 50% above non-
pregnant values.
2- The median respiratory rate is increased in the third trimester.
3- A plasma creatinine of 0.6 mg/dL and a plasma urea of 15 mg/dL is within
normal limits in pregnancy.
4- Maternal weight gain is a good predictor of a small for gestation age (SGA)
infant.
5- Maximum maternal weight gain occurs between 20 and 24 weeks' gestation.

Directions: In the following questions, each item has 5 lettered options each of which
could be a true or a false statement. Tick each of these options according to whether
they are false or true

481- The following statement(s) regarding intrapartum cardiotocography is (are)


correct:
a) It is useful in high risk pregnancy.
b) It is a relation between fetal heart rate and fetal kicks.
c) Reactive trace means a reassuring test.
d) 80% of abnormal test pattern in the absence of fetal hypoxia.
e) 90% of early decelerations will need cesarean section

482- The following statement(s) regarding fetal heart rate is (are) correct:
a) Normal baseline is between 110-150.
b) Acceleration means increase in fetal heart rate more than 10 beats/min lasting less
than 10 seconds.
c) Fetal Bradycardia means a rate less than 100 b/min for 15 min.
d) Loss of beat to beat variability is a poor sign.
e) may be increased in maternal thyrotoxicosis

483- The following statement(s) regarding Obstetrical ultrasound is (are) correct:


a) After 30 weeks gestation, it is more accurate than Naegele's rule in calculating the
EDD.
b) Transabdominal transducer has a frequency more than 10 MHz.
c) It can differentiate between types of abortion.
d) It is useful in the follow up of intrauterine contraceptive device (IUCD).
e) It is useful in the management of antepartum hemorrhage.

484- Study of the blood flow velocity in the umbilical arteries by Doppler ultrasound
shows:
a) It increases with advancing gestational age.
b) It decreases in cases of intrauterine growth restriction.
c) It increases with maternal hypertension.
d) It increases with placental insufficiency.
e) Abnormal changes in twin-to-twin transfusion.
NORMAL LABOR
Directions: Each of the numbered items or incomplete statements in this section is
followed by answers or by completions of the statement. Select the ONE lettered answer
or completion that is BEST in each case.

485- Cephalopelvic disproportion in the absence of gross pelvic abnormality can be


diagnosed by:
a) Ultrasound.
b) A maternal stature of <155 cm.
c) Trial of labor.
d) X-ray pelvimetry.
e) Pelvic examination.

486- The cardinal movements of labor and delivery involve a sequence of events that
occurs in an orderly fashion. Which of the following sequences is correct?
a) Descent, internal rotation, flexion.
b) Engagement, flexion, descent.
c) Engagement, internal rotation, descent.
d) Engagement, descent, flexion.
e) Descent, flexion, engagement.

487- Regarding asynclitism, the INCORRECT statement is:


a) In the anterior parietal bone presentation, the sagittal suture lies posteriorly.
b) In the posterior type, the posterior parietal bone has passed the sacral promontory.
c) In the posterior type, the anterior parietal bone has passed the symphysis pubis.
d) The posterior type is more common in primigravidas while the anterior type is more
common in multigravidas.
e) Asynclitic head is more easily engaged than synclitic head.

488- Asynclitism is best defined as:


a) Flexion of the descending fetal head from pelvic floor resistance.
b) Failure of the sagittal suture to lie exactly midway between the symphysis ad sacral
promontory.
c) Failure of the descent because of inadequate uterine contractions.
d) Inability of the fetal head to pass through the pelvic inlet.
e) Inability of internal rotation after the fetal head has reached the ischial spines.

489- Caput succedaneum:


a) Resolves spontaneously after labor.
b) May lead to jaundice of the baby postpartum.
c) Indicates a traumatic vaginal delivery.
d) Indicates that the fetal head is engaged.
e) Is a sign of intrauterine fetal death.

490- Engagement is said to have occurred when which of the following events takes
place?
a) The infant's head is within the pelvis.
b) The biparietal diameter of the infant's head is through the plane of the inlet.
c) The presenting part is just above the level of the ischial spines.
d) The vertex is in the transverse position.
e) The infant's head is flexed.

491- A woman delivers a 4.5 kg infant with a midline episiotomy and suffers a third-
degree tear. Inspection shows that which of the following structures is intact?
a) Anal sphincter.
b) Perineal body.
c) Perineal muscles.
d) Fascia.
e) Rectal mucosa.

492- The INCORRECT statement about engagement is:


a) The biparietal diameter (BPD) passes the plane of pelvic inlet in cephalic
presentation.
b) The lowest part of the head is at station -1.
c) Short cord is one of causes of non-engagement.
d) Tn multipara, it may occur only at the 2nd stage of labor.
e) Can be diagnosed by the rule of fifths.

493- Primigravida, 24 years-old, in labor for 16 hours and the cervix is arrested at 9 cm
for 3 hours, position is ROP, station 0 and molded. There is fetal late decelerations
over the last 30 minutes. Delivery is BEST managed by:
a) Forceps rotation and then traction.
b) Ventouse (vacuum) extraction.
c) Craniotomy then forceps.
d) Lower segment cesarean section (LSCS).
e) Upper segment cesarean section (USCS).

494- Primigravida with a fully dilated cervix for 2 hours, head station = -1 with molding
and diffuse caput can be safely delivered by:
a) Cesarean section.
b) Forceps delivery.
c) Vacuum extraction.
d) Internal podalic version then breech extraction.
e) Fundal compression with deep episiotomy.

495- Which of the following is NOT a characteristic of active-phase uterine


contractions?
a) They create 40 mm Hg of pressure.
b) They cause dilation of the cervix.
c) They cause thickening of the lower uterine segment.
d) They occur-every 2 to 4 minutes.
e) They last for 45 seconds.

496- Normal labor includes the following:


a) Delivery of a 39 weeks baby.
b) Delivery by mid forceps.
c) Breech delivery.
d) Twin pregnancy delivered vaginally.
e) Induction of labor by oxytocin.

497- The following statements regarding true labor pains are correct EXCEPT:
a) They are regular.
b) They are relieved by sedation or enema.
c) They are associated with cervical dilatation.
d) They may be associated with ROM.
e) They increase in intensity by time.

498- A 16-year-old primigravida presents with severe preeclampsia. She has meconium-
stained amniotic fluid. Contractions occur every 3 minutes, and there is a late
deceleration with each contraction. Fetal changes would include which of the
following?
a) Decreased lactic acid.
b) Increased pH.
c) Increased PCO2.
d) Decreased bicarbonate.
e) Increased PO2-

499- A woman arrives at the hospital in active labor stating that her membranes
ruptured 2 hours earlier. The fetal heart rate (FHR) monitor shows decelerations
that do not seem related to any point in the contraction and quickly return to
baseline with good variability. Which of the following is characteristic of these
decelerations?
a) They are associated with fetal head compression.
b) They are caused by uteroplacental insufficiency.
c) They are indications of fetal metabolic acidosis.
d) They are associated with a rise in fetal pH.
e) They are influenced by the ruptured-membrane status.

500- A patient in the active phase of labor is 5-cm dilated, and the fetal heart monitor
shows decreased variability. The monitor shows a baseline fetal rate of 125 beats
per minute (bpm), with occasional late decelerations. A fetal scalp pH of 7.22 is
obtained. This situation indicates which of the following conditions or actions?
a) Significant fetal metabolic acidosis.
b) The need to repeat the fetal scalp pH test in 20 minutes.
c) The need for immediate cesarean section
d) Chorioamnionitis with maternal fever.
e) The need for an intrauterine pressure catheter.

501- A 31-year-old woman has been pushing in the second stage of labor for 2 hours.
The vertex is at the +2 station. Each contraction is associated with a fetal
bradycardia as low as 100 beats per minute (bpm) that lasts for 30 seconds. This
clinical scenario suggests which of the following situations?
a) Systemic fetal hypoxia.
b) Poor fetal outcome.
c) An association with oligohydramnios.
d) Fetal head compression.
e) A depressed fetal pH.

502- Which of the following fetal mechanisms does NOT compensate for the normal
low fetal arterial partial pressure of oxygen (PO2)?
a) Increased fetal cardiac output.
b) Increased fetal systemic blood flow rates.
c) Increased fetal pulmonary blood flow.
d) Increased affinity of fetal blood for oxygen.
e) Increased fetal oxygen-carrying capacity.

503- Which of the following explanations is NOT an explanation for decreased


variability to the fetal heart rate (FHR) tracing?
a) Fetal "sleep state".
b) Prematurity.
c) Barbiturate ingestion.
d) Fetal stimulation.
e) Asphyxia.

504- Which of the following is NOT a characteristic or associated finding with late
decelerations?
a) They are seen in patients with preeclampsia.
b) They may be associated with respiratory alkalosis.
c) They are associated with a decreased uteroplacental blood flow.
d) They usually are accompanied by a decrease in partial pressure of oxygen
(PO2).
e) They usually are accompanied by an increase in partial pressure of carbon dioxide
(PCO2).

505- A woman with ruptured membranes is in the active phase of labor and is 5-cm
dilated with sustained, deep variable decelerations. The decision is made to
perform a cesarean section. Which of the following would NOT be an
appropriate intrauterine resuscitative measure done before the cesarean section?
a) Increase the intravenous fluids.
b) Place the patient in the supine position.
c) Start nasal oxygen.
d) Start amnio-infusion.
e) Administer subcutaneous terbutaline.

506- A 25-year-old pregnant woman presents to the labor floor reporting extreme
abdominal pain and heavy vaginal bleeding. The fetal heart monitor shows a
fetal bradycardia with late decelerations. Which of the following acid-base
characteristics would NOT be present in the fetus?
a) Respiratory alkalosis.
b) An increased partial pressure of carbon dioxide (PCO2).
c) A drop in fetal pH.
d) Accumulation of lactic acid.
e) Metabolic acidosis.
507- Epidural anesthesia:
a) Does not affect uterine activity.
b) Is contraindicated in patients with heart valve lesions.
c) Should be routinely administered during the first stage of labor.
d) Increases the risk of postpartum hemorrhage.
e) Should be offered to highly selected cases in labor.
508- The following statements regarding ischial spines are correct EXCEPT:
a) They make the beginning of the forward curve of the pelvis.
b) They are landmarks for pudendal nerve block procedure.
c) They indicate a normal pelvis when particularly prominent.
d) They help to assess station of the presenting part.
e) They lie at the level of the plane of least pelvic dimensions.

Directions Questions 509 through 524: Each set of matching questions in this section
consists of a list of 4 to 8 lettered options followed by several numbered items. For each
item, select the ONE best lettered option that is most closely associated with it. Each
lettered heading may be selected once, more than once, or not at all.

Questions 509-519
Match the diameter given with the corresponding description
a) 9 cm
b) 9.5 cm
c) 10.5cm
d) 11 cm
e) 11.5 cm
f) 12cm
g) 12.5 cm
h) 13.5 - 13.75 cm
509- Right oblique diameter of the pelvic inlet
510- Left oblique diameter of the pelvic inlet
511- True conjugate of pelvic inlet
512- Obstetric conjugate
513- Sacrocotyloid diameter
514- Bispinous diameter
515- Bituberous diameter
516- Biparietal diameter
517- Mentovertical diameter
518- Suboccipito bregmatic diameter
519- Occipitofrontal diameter

Questions 519-524
Match the appropriate terra with the description given:
a) Lie
b) Station
c) Denominator
d) Position
e) Presentation

520- Left occipito anterior


521- Mentum
522- Longitudinal
523- Vertex
524- Zero

Directions: Each of the following numbered items or incomplete statements in this


section is followed by numbered items that in certain combination leads to the BEST
lettered combined answer or/and completion. According to the following directions,
choose the BEST lettered answer:
If
1,2,3 are correct, choose A
1,3 are correct, choose B
2,4 are correct, choose C
1,2,3,4 are correct, choose D
All are correct, choose E

525- The following statements relating to ischial spines are correct:


1 - They lie between the greater and lesser sciatic notches.
2- They mark the beginning of the forward curve of the birth canal.
3- The internal pudendal nerve lies in close relation to the spines.
4- When the widest transverse diameter of the fetal skull is at the level of the spines,
the head is engaged.
5- They are of particular prominence in the normal female pelvis.

526- An anthropoid pelvis:


1 - Is of good obstetric value.
2- Is associated with deep transverse arrest.
3- Has a narrow sub pubic arch.
4- Has a male-shaped brim.
5- Is associated with spondylolisthesis.

527- Fetal presentation:


1 - Is the part of the fetus that enters the pelvis first.
2- In face presentation the occiput and back comes into contact.
3- Is usually cephalic,
4- The denominator is the lowest part of the presenting area.
5- In cephalic presentation, it is more common to be face.

528- The following statements regarding occipito anterior presentations are correct:
1 - The back of the fetus is parallel to the back of the mother.
2- Left occipito anterior presentation the head descends in the left oblique diameter.
3- The head of the fetus is fully flexed.
4- The back of the fetus is directed posteriorly.
5- They are less common than occipitoposterior.

529- Engagement of the head can be assessed by:


1 - Abdominal examination.
2- Relationship to ischial tuberosity.
3- Relationship to ischial spines.
4- Degree of molding.
5- Degree of caput formation.

Directions: In the following questions, each item has 5 lettered options each of which
could be a true or a false statement. Tick each of these options according to whether
they are false or true

530- Regarding the fetal skull fontanelles:


a) There are only 2 fontanelles.
b) The posterior fontanelle is closed at 28 wks G.A.
c) The anterior fontanelle is closed at birth.
d) Neonatal brain U/S is done usually through the posterior fontanelle.
e) The anterior fontanelle is irregular in shape.
531- Placental separation:
a) Often occurs by Schultz method.
b) Duncan method is more liable for retained parts.
c) The earliest sign is: the uterus becomes more globular, smaller and harder,
d) Normally takes 1 -2 hours in primigravida (PG).
e) Can be diagnosed by stop of pulsation of the cord.

532- First stage of labor:


a) Lasts for maximum 8 hours in primigravida (PG).
b) Starts with true labor pains & ends after delivery of the fetus.
c) Latent phase is the first phase of this stage.
d) Includes cervical dilatation & effacement.
e) Active phase is characterized by increased rate of cervical dilatation and descent of
the presenting part.

533- The first stage of labor:


a) Ends in expulsion of the fetus.
b) Begins when membranes rupture.
c) Can be shortened with the use of oxytocin.
d) Normally lasts for more than 24 hours in a primigravida.
e) Is prolonged in malposition of the head.
NORMAL AND ABNORMAL PUERPERIUM
Directions: Each of the numbered items or incomplete statements in this section is
followed by answers or by completions of the statement. Select the ONE lettered answer
or completion that is BEST in each case.

534- The following statements regarding maternal mortality are correct EXCEPT:
a) It is defined as "the death of any woman due to any cause whilst pregnant or within
42 days of the TOP, irrespective of the duration and the site of the pregnancy; from
any cause related to or aggravated by the pregnancy or its management but not from
accidental or incidental causes.
b) In Egypt it is 10 per 100.000 live births.
c) Pulmonary Embolism is the most important cause in developed countries
d) Maternal mortality rate is affected by maternal age, parity & mode of delivery.
e) It can be reduced by proper antenatal care.

535- Maternal mortality refers to the number of maternal deaths that occur as the
result of the reproductive process per:
a) 1000 births.
b) 10.000 births.
c) 100.000 births.
d) 10.000 live births.
e) 100.000 live births.

536- Which of the following is the INCORRECT statement regarding postpartum


mastitis:
a) The most common pathogenic organism is staphylococcus aureus,
b) It usually occurs within the first postpartum week.
c) The offending organism could be cultured from breast milk.
d) The mother was probably a chronic carrier of the organism prior to breast feeding.
e) There is no contraindication to the continuance of breast feeding.

537- The following statement about birth control after delivery is correct:
a) It is not important until after the first menses.
b) It is not necessary in a woman who is nursing.
c) It should begin immediately in a non-lactating woman.
d) It is not necessary for 3 months after a cesarean section
e) The best method is intrauterine device in > 90% of cases.
ABNORMAL PREGNANCY
Directions: Each of the numbered items or incomplete statements in this section is
followed by answers or by completions of the statement. Select the ONE lettered answer
or completion that is BEST in each case.

538- The following is associated with increased abortion rate EXCEPT:


a) Tobacco.
b) Hyperprolactinemia prior to pregnancy.
c) Radiation over 5 rads.
d) Coffee in excess of 4 cups per day.
e) Class A1 diabetes mellitus.

539- The major cause of first trimester abortion is:


a) Chromosomal anomalies.
b) Anatomic defects of the uterus or cervix.
c) Circumvallate placentation.
d) Endocrine factors.
e) Immunological factors.

540- Subacutely disturbed tubal pregnancy can be excluded if:


a) Menstrual pattern is normal.
b) No pelvic pain.
c) 1UCD is intrauterine.
d) Transvaginal ultrasound is normal.
e) Serum βhCG is negative.

541- Most common site of occurrence ectopic pregnancy is:


a) Isthmic or interstitial portion of the Fallopian tube.
b) Corneal end of the tube.
c) Ampullary portion of the fallopian tube.
d) Fimbrial end of the Fallopian tube.
e) Ovary.

542- Contraindication to medical therapy (Methotrexate) in tubal pregnancy is:


a) Ectopic size 3 cm or less.
b) Desire for future fertility.
c) History of active hepatic or renal disease.
d) A sonographically non viable pregnancy.
e) Absence of active bleeding.

543- The risk factors for an ectopic pregnancy include the following EXCEPT:
a) History of pelvic inflammatory disease.
b) Previous ectopic pregnancy.
c) Use of combined oral contraceptives.
d) Assisted reproductive techniques.
e) 1UD use.

544- The following statements regarding ectopic pregnancy are correct EXCEPT:
a) Accounts for 10% of all maternal mortality.
b) There has been a four fold increase over the past 20 years.
c) The majority of cases (over 95%) occur in the tube.
d) The incidence of ectopic is directly related to the prevalence of salpingitis.
e) The effect of ectopic pregnancy on the future fertility is insignificant.

545- Diagnosis of ectopic pregnancy can be ruled out in almost all clinically significant
cases if:
a) Transvaginal sonography showed no abnormalities in the adnexal region.
b) Serum β-hCG was negative.
c) Patient did not miss her period.
d) Patient is using the IUCD for contraception.
e) There is no pelvic fluid by transvaginal uterine ultrasound.

546- The underlying pathogenic process in hydatidiform mole is:


a) Androgenesis.
b) Parthenogenesis.
c) Parent mutation.
d) Normal fertilization.
e) Non disjunction.

547- The following statements regarding hydatidiform moles are correct EXCEPT:
a) Usually have female karyotype.
b) may be complicated by thyrotoxicosis.
c) may be complicated by ovarian cysts.
d) a feto-protein is a good marker of disease post-evacuation.
e) Is more prevalent in the advanced age group.

548- The following statements regarding vesicular mole are correct EXCEPT:
a) A fetus may coexist in utero.
b) Invasive mole penetrates the myometrium and has identifiable chorionic villi.
c) Commonly comprises cells with 46XX chromosomes of paternal origin.
d) Has a greater than 20-fold increase in incidence in mothers over 45 years old.
e) After evacuation of a hydatidiform mole, the patient is monitored using β-hPL
assays.

549- The following statements regarding gestational trophoblastic disease are true
EXCEPT:
a) Hydatidiform moles arise from paternal genetic material.
b) Choriocarcinoma is characterized by an absence of chorionic villi.
c) There is a high incidence in South- East Asia.
d) Choriocarcinoma is highly chemosensitive with methotrexate as the agent most
frequently used.
e) Over 80% of patients with hydatidiform mole have β-hCG levels that return to
normal by 2 weeks after suction evacuation.

550- Risk factors for persistent trophoblastic disease following molar evacuation
include the following EXCEPT:
a) Young age (<20 years old).
b) Old age (> 40 years old).
c) Increased uterine size.
d) Presence of theca lutein cysts.
e) β-hCG level > 100,000 mlU/ml.
551- The best contraceptive method for patients with gestational trophoblastic diseases:
a) Tubal ligation.
b) Hysterectomy.
c) Combined oral contraceptive pills.
d) Intra-uterine contraceptive device.
e) Subcutaneous implants.

552- The risk of recurrence of molar and or trophoblastic disease in subsequent


pregnancies is:
a) 2%.
b) 5%.
c) 8%.
d) 10%
e) 15%.

553- The following is the main difference between complete and partial vesicular mole:
a) Propensity of metastases.
b) Triploidy & diploidy.
c) Need for chemotherapy.
d) Trophoblastic hyperplasia.
e) Androgenesis.

554- The most common site of metastatic disease in choriocarcinoma is:


a) Brian.
b) Liver.
c) Vagina.
d) Lung.
e) Ovary.

555- Theca lutein cyst are characterized by the following EXCEPT:


a) Are best treated conservatively.
b) Are best treated surgically.
c) Are seen occasionally in normal pregnancy.
d) Are seen in ovarian hyperstimulation syndrome.
e) Are liable to complications including hemorrhage, rupture and torsion.

556- The following are risk factors for the development of placental abruption
EXCEPT:
a) Smoking.
b) Folic acid deficiency.
c) Pre-eclampsia.
d) History of threatened abortion.
e) Previous placental abruption.

557- The percentage of patients with a history of prematurity who will have another
premature infant is:
a) 0% to 10%.
b) 20% to 30%.
c) 40% to 50%.
d) 60% to 70%.
e) 80% to 90%.

558- A 16-year-old primigravida reports that she is experiencing regular menstrual


cramping every 2 minutes. She is 28 weeks pregnant. After taking a history, the
first thing that the physician should do is:
a) Send her to the labor floor immediately.
b) Confirm the frequency of contractions by abdominal palpation.
c) Evaluate fetal well-being with a fetal monitor.
d) Evaluate the cervix by speculum examination.
e) Immediate cesarean section.

559- The most common cause of vaginal bleeding complicating premature labor is:
a) A vaginal laceration.
b) An endocervical polyp.
c) Cervical dilation.
d) Placenta previa.
e) Placental abruption.

560- A 39-year-old former Olympic athlete (gravida 5, para 0311) comes for her first
prenatal visit at 8 weeks' gestation. After undergoing a prenatal genetic counseling
session, she asks her physician about current recommendations concerning her
risk of having a premature infant. The physician should explain that:
a) She is at no risk for premature delivery.
b) She can continue her plans for running a marathon in 1 month.
c) Her age has no bearing on this pregnancy.
d) He would like to see her more often to perform cervical, examinations.
e) He would like to see her more often only if any symptoms of premature labor
develop.
561- Which of the following is a sign of late premature labor?
a) Increased vaginal discharge.
b) Increased uterine contractions.
c) Low back pain.
d) Cervical dilation, to 4 cm.
e) Worsening pelvic pressure.
562- Which of the following is NOT a predisposing factor for premature labor?
a) Maternal age older than 30 years.
b) Smoking more than 10 cigarettes per day.
c) Exposure to diethylstilbestrol (DES) in utero with a documented uterine structural
abnormality.
d) Multiparity with more than four previous deliveries.
e) Twin gestation.

563- Which of the following drugs is NOT used to inhibit premature labor?
a) Ethanol.
b) Magnesium sulfate.
c) Phenobarbital.
d) Ritodrine.
e) Terbutaline.
564- The first step in the assessment of the postterm gestation is:
a) Ultrasound examination.
b) Determination of the true length of gestation.
c) Measurement of fetal heart rate (FHR).
d) Determination of amniotic fluid volume.
e) Contraction stress test.

565- The perinatal mortality rate at 44 weeks' gestation is:


a) Less than 1 %.
b) 1 % to 2%.
c) 2% to 3%.
d) 4% to 5%.
e) 6% to 7%.

566- After having identified a fetus at risk in prolonged pregnancy, management should
consist of:
a) Amniocentesis for maturity studies.
b) Delivery regardless of the status of the cervix.
c) Fetal sampling of scalp pH.
d) Measurement of human chorionic somatomammotropin.
e) Repeat antepartum studies in 1 week.

567- Which of the following is NOT included in the biophysical profile for the risk
assessment of a postterm fetus?
a) Fetal breathing.
b) Amniotic fluid volume.
c) Fetal tone.
d) Contraction stress test (CST).
e) Fetal motion.

568- A woman who is 16 weeks pregnant has a maternal serum a-fetoprotein (MSAFP)
level of 2.8 multiple of the mean (MOM). Which of the following conditions is
NOT an explanation of this abnormal finding?
a) Anencephaly.
b) Down syndrome.
c) Duodenal atresia.
d) Omphalocele.
e) Twins.

569- A 24-year-old woman is in a car accident and is taken to an emergency room where
she receives a chest x-ray and film of her lower spine. It is later discovered that
she is 10 weeks pregnant. She should be counseled that:
a) The fetus has received 50 rads.
b) Either chorionic villus sampling (CVS) or amniocentesis is advisable to check for
fetal chromosomal abnormalities.
c) At 10 weeks the fetus particularly susceptible to derangements of the central nervous
system.
d) The fetus has received rads below the assumed threshold for radiation damage.
e) The accident may cause placental abruption

570- The following statements regarding biochemical screening for Down's Syndrome
are correct EXCEPT:
a) Has been shown to improve detection rates compared to maternal age alone.
b) Diagnosis by chorionic villus sampling (CVS) is one component of the triple test.
c) Is most strongly dependent on maternal serum a feto-protein (MSAFP).
d) A booking ultrasound examination is required prior to biochemical screening.
e) Parental karyotype analysis is indicated if there is a family history of Down's
Syndrome.

571- A pregnant woman is routinely screened for which of the following disease
entities?
a) Parvovirus.
b) Toxoplasmosis.
c) Cytomegalovirus (CMV).
d) Syphilis.
e) Herpes simplex virus (HSV).

572- The following statements regarding toxoplasmosis in a pregnant woman are


correct EXCEPT:
a) It can be acquired by eating infected raw meat.
b) It can be acquired by contact with cat feces.
c) Infection in early pregnancy may lead to abortion.
d) It is diagnosed by the presence of IgG.
e) Fetal affection may occur transplacentally.

573- Which of the following is a common reservoir for toxoplasmosis?


a) School-age children.
b) Bird droppings.
c) Cats.
d) Contaminated seafood.
e) Blood.

574- Which of the following factors is NOT associated with an increased risk of
perinatal morbidity?
a) Low socioeconomic status.
b) Low maternal age (less than 20 years old).
c) Heavy cigarette smoking.
d) Alcohol abuse.
e) Exercise.

575- Maternal age younger than 20 years does NOT increase the risk for which of the
following conditions?
a) Low birth weight.
b) Fetal death.
c) Uterine dysfunction.
d) A genetically abnormal conceptus.
e) Premature delivery.
576- The following complications could frequently affect the elderly primigravida
EXCEPT:
a) Gestational diabetes mellitus.
b) Pre-eclampsia.
c) Precipitate labor.
d) Vesicular mole.
e) Placental abruption.

Directions Questions 577 through 581: Each set of matching questions in this section
consists of a list of 4 to 8 lettered options followed by several numbered items. For each
item, select the ONE best lettered option that is most closely associated with it. Each
lettered heading may be selected once, more than once, or not at all.
a) Conservative treatment
b) Allowance or trial of vaginal delivery
c) Elective cesarean section
d) Emergency cesarean section
e) Cesarean hysterectomy
577- Patients with placenta previa with severe bleeding & immature fetus
578- Patient with placenta previa centralis accidentally discovered at 37 weeks plus 4
days gestation.
579- Patient with mild placental abruption, no fetal distress at 33 weeks gestation
580- Patient with severe accidental hemorrhage, intrauterine fetal death, cervix is 8 cm
dilated
581- Patient with placental edge at 3 cm from the cervical internal os, presenting in
active labor

Directions: Each of the following numbered items or incomplete statements in this


section is followed by numbered items that in certain combination leads to the BEST
lettered combined answer or/and completion. According to the following directions,
choose the BEST lettered answer:
If
1,2,3 are correct, choose A
1,3 are correct, choose B
2,4 are correct, choose C
1,2,3,4 are correct, choose D
All are correct, choose E

582- Medical treatment of ectopic pregnancy could be tried if:


1- Serum p-hCG is <3000 mill/ml.
2- There is sonographically viable pregnancy.
3- Patient is hemodynamically stable.
4- Ectopic sac is >4 cm in diameter.
5- Ectopic pregnancy is disturbed.

583- The following statements regarding recurrent (habitual) abortion are true:
1 - The most common hormonal cause in habitual abortion is progesterone
deficiency.
2- No etiological factor is identified in approximately 50%.
3- The incidence is 1% of abortions.
4- It is defined as 3 consecutive induced abortions.
5- The most common genetic abnormality in habitual abortion is polyploidy.

584- The following statements regarding missed abortion are true:


1 - It implies that despite fetal death, the pregnancy has been retained.
2- Ultrasound is helpful in its diagnosis,
3- Milk secretion may start spontaneously from the breast.
4- There may be brownish vaginal discharge.
5- There are exaggerated pregnancy symptoms.

585- As regards placenta previa, the following are true:


1 - It is manifested by painless bleeding.
2- The initial hemorrhage is usually fatal.
3- It may predispose to postpartum hemorrhage.
4- Its incidence is unaffected by parity.
5- Its incidence is decreased with maternal age.

586- The following statements regarding the diagnosis of early ectopic pregnancy are
true:
1- Transvaginal ultrasound is helpful in conjunction with hCG in diagnosis of early
ectopic.
2- Clinical examination is almost diagnostic of ectopic.
3- Laparoscopy is the golden standard for diagnosis in doubtful cases.
4- β-subunit hCG is doubled every 48 hours.
5- Culdocentesis is a current method for early diagnosis.

587- As regards the management of ectopic pregnancy, the following are true:
1 - The majority of undisturbed cases can be treated laparoscopically.
2- Cervical pregnancy may need hysterectomy.
3- Few ectopic pregnancies can resolve spontaneously.
4- Salpingectomy is the only option for surgical management.
5- Laparoscopy is preferred in shocked patients.

Directions: In the following questions, each item has 5 lettered options each of which
could be a true or a false statement. Tick each of these options according to whether
they are false or true

588- Conservative management of ectopic pregnancy by intratubal injection of


methotrexate:
a) Is contraindicated in the presence of tubal rupture.
b) Should be limited to cases with a tubal diameter of 3-4 cm.
c) Is particularly effective when ultrasound examination demonstrates fetal cardiac
activity.
d) Should be followed by serial measurement of serum progesterone until values fall to
the non-pregnant range.
e) Is safe provided that citrovorum factor is administered concurrently.

589- As regards vasa previa:


a) It is a common condition.
b) It occurs with circumvallate placenta.
c) It is the only cause of antepartum hemorrhage of fetal origin.
d) The fetal mortality is 50-75%.
e) It can be treated by a conservative way.

590- Complications of abruptio placentae include:


a) Hemorrhagic shock.
b) Consumptive coagulopathy.
c) Acute renal failure is common in the mild form.
d) Maternal mortality rate of 10%.
e) IUFD in all cases if less than ½ of the placenta is separated.

591- A Couvelaire uterus:


a) Is an indication for hysterectomy.
b) may contract well with stimulation.
c) Results from excessive oxytocin.
d) Requires fibrinogen therapy.
e) Complications of severe abruption.
MEDICAL DISORDERS WITH PREGNANCY

Directions: Each of the numbered items or incomplete statements in this section is


followed by answers or by completions of the statement. Select the ONE lettered answer
or completion that is BEST in each case.

595- Which of the following is NOT an independent risk factor for pregnancy-induced
hypertension (PIH)?
a) Multiple gestation.
b) Chronic hypertension.
c) African-American race.
d) Age younger than 20 years.
e) Diabetes.
596- Which of the following is NOT a sign of severe pregnancy-induced hypertension?
a) Oligohydramnios.
b) Proteinuria in excess of 1 g/24 hours.
c) Thrombocytopenia.
d) Elevated serum creatinine.
e) Elevated transaminases.

597- In eclampsia:
a) Peripheral edema is common.
b) The circulating blood volume is increased.
c) Immediate delivery under general anesthesia is the management of choice.
d) Beta blockers would be the best antihypertensive therapy.
e) Delivery is contraindicated before 34 weeks.

598- Which of the following is a sign of heart disease in pregnancy?


a) Lower extremity edema.
b) Systolic murmur.
c) Increased respiratory effort.
d) Arrhythmia.
e) Dyspnea.
599- Physiologic anemia of pregnancy is due to:
a) Increase iron requirements.
b) Increase folic acid requirements.
c) Disproportionate increase in plasma volume compared with RBC mass.
d) Repeated blood loss.
e) All of the above.
601- Folic acid deficiency during pregnancy is associated with the following EXCEPT:
a) Hyper-segmented neutrophils.
b) Megaloblastic erythropoiesis.
c) Increased infections.
d) Megaloblastic anemia.
e) Abruptio placentae.
602- In the metabolism of iron, the following statement are correct EXCEPT:
a) Absorption occurs mainly in the duodenum and upper jejunum.
b) Absorption is increased when stores are depleted.
c) Traces of copper are essential for hemopoiesis.
d) Transferrin is increased in pregnancy.
e) Ferric salts are more readily absorbed than ferrous salts.
603- A pregnant woman with the sickle cell trait is at risk for an increased incidence of
which of the following?
a) Perinatal mortality.
b) Low-birth weight infants.
c) Pregnancy-induced hypertension.
d) Urinary tract infection.
e) Spontaneous abortion.
604- The following pregnancy related phenomena may have immunological etiological
basis EXCEPT:
a) Recurrent fetal losses.
b) Pre-eclampsia.
c) Neonatal jaundice.
d) Ectopic pregnancy
e) Transient neonatal thyrotoxicosis.
605- The adverse pregnancy outcomes in Antiphospholipid Antibodies Syndrome may
include the following EXCEPT;
a) IUGR.
b) Early onset pre-eclampsia.
c) Placental abruption.
d) Polyhydramnios.
e) Arterial and venous thrombosis.

606- At 30 weeks gestation amniocentesis is performed to Rh negative isoimmunized


patient (having anti D titer of 1:126) the AOD is placed high in zone II on chart.
Which of the following measures should be done:
a) Repeat maternal serum antibody titer.
b) Repeat U/S examination and amniocentesis after 1 week.
c) Perform an immediate intrauterine transfusion.
d) Prepare for pregnancy termination.
e) Perform RH typing of her husband.

607- In iron deficiency anemia in pregnancy, the following is true:


a) Mean corpuscular hemoglobin content and mean corpuscular concentration are both
low.
b) Mean corpuscular volume is raised.
c) Blood transfusion is indicated if HB level fall below 9 gm%.
d) There is usually a chronic blood loss causing the anemia.
e) There is an increased risk of preeclampsia.

608- During pregnancy a woman needs additional iron to satisfy demands of fetus,
placenta, and her own increasing hemoglobin mass total ante partum iron extra
needs is approximately:
a) 60 mg
b) 250 mg.
c) 600 mg.
d) 1350mg
e) 1900mg

609- In asymptomatic bacteriuria:


a) It is more common in non-pregnant than pregnant women.
b) Diagnosis depends on complex of signs.
c) Follow up only is satisfactory initial management.
d) Enterobacter species is the most common causative organism.
e) Further urine culture is needed for follow up.

610- Management of asymptomatic bacteriuria includes:


a) Expectant management.
b) Induction of labor.
c) Antibiotics.
d) Diuretics.
e) Intravenous hydration.

611- Which of the following factors does NOT contribute to an acute urinary tract
infection during pregnancy, delivery, and the puerperium?
a) Compression of the ureter by the large uterus at the pelvic brim.
b) Increased ureteral tone and peristalsis.
c) Asymptomatic bacteriuria.
d) Decreased bladder sensitivity after epidural anesthesia.
e) Bladder catheterization following delivery.

612- Which of the following is NOT a true statement about urinary tract infections
during pregnancy?
a) Asymptomatic bacteriuria in pregnancy needs to be treated.
b) Pregnancy increases the risk for development of asymptomatic bacteriuria.
c) Acute systemic pyelonephritis resulting from asymptomatic bacteriuria is associated
with premature labor and delivery.
d) Few women who are not bacteriuric at their first prenatal visit develop asymptomatic
bacteriuria.
e) The incidence of asymptomatic bacteriuria is higher in pregnant women with a low
socioeconomic status and increased parity and age.

613- During pregnancy, the renal glomerular filtration rate (GFR) can increase by as
much as:
a) 10 percent.
b) 25 percent.
c) 50 percent.
d) 75 percent.
e) 100 percent.

614- The followings are known to cause acute renal failure EXCEPT:
a) HELLP syndrome.
b) Sickle cell trait.
c) Septicemia.
d) Concealed antepartum hemorrhage.
e) Incompatible blood transfusion.

615- Obstetrical causes of acute renal failure include the following EXCEPT:
a) Septic abortion.
b) Eclampsia.
c) HELLP syndrome.
d) Placenta circumvallate.
e) Abruptio placentae.

616- Medical history suggestive of diabetes in pregnancy include the following


EXCEPT:
a) Past history of macrosomia.
b) Past history of eclampsia.
c) Past history of congenital malformations.
d) Past history of unexplained stillbirths.
e) Positive family history.

617- Which of the following is significant in the management of pregnant diabetic:


a) Stating oral hypoglycemic agents.
b) Cesarean section before 36 weeks gestational age.
c) Maintenance of one hour post-prandial blood glucose level below 140 mg/dl.
d) Bed rest.
e) Carbohydrate free diet

618- Effects of pregnancy on diabetes mellitus include the following EXCEPT:


a) Pregnancy has triphasic effects on glucose metabolism.
b) Control of the blood sugar becomes more difficult.
c) Insulin dosage usually increased after 20 weeks gestational age.
d) Increased liability to hypoglycemia.
e) Progression of chronic diabetic complications may be accelerated.

619- Pre-conceptional management of diabetic patient includes the following EXCEPT:


a) Laboratory estimation of glycosylated hemoglobin.
b) Achievement of euglycemia.
c) Ultrasound examination.
d) Discontinuation of oral hypoglycemic agents.
e) Assessment of diabetic complications.

620- A 1-hour glucose tolerance test in a woman with a previous stillborn infant re-
suited in the following value: 1-hour blood sugar = 140. Follow-up for this patient
should include which of the following?
a) Nothing further.
b) A 2000-calorie diet.
c) Standard glucose tolerance test.
d) Home glucose urine testing.
e) A 2-hour postprandial blood sugar.

621- The infant of a diabetic mother is NOT at risk for which of the following?
a) Increased perinatal death rate.
b) Hypocalcemia.
c) Hyperglycemia.
d) Neural tube defects.
e) Macrosomia.
622- The following statements regarding congenital malformations in babies of diabetic
mothers are correct EXCEPT:
a) Cause half the perinatal deaths.
b) Are most often skeletal abnormalities.
c) Are common if control is poor in the first trimester.
d) Have fallen significantly in the last 30 years.
e) Are commoner in insulin-dependent than non-insulin-dependent mothers.

623- Smoking during pregnancy may lead to all the following EXCEPT:
a) Spontaneous abortion.
b) Preterm labor.
c) Placenta previa.
d) Fetal macrosomia.
e) Placental abruption.
624- Treatment of bronchial asthma during pregnancy include all the following
EXCEPT:
a) Betamethazone.
b) Ipratropium.
c) Beta blocking agents.
d) Theophylline.
e) Disodium cromoglycate.
625- Heparin therapy in thromboembolic diseases:
a) Osteoporosis are more common with unfractionated heparin (UFH) treatment.
b) Thrombocytopenia does not occur except after 1 week of heparin treatment.
c) Over dose is the major risk of heparin treatment.
d) Long term heparin treatment often results in thrombocytopenia.
e) Unfractionated heparin (UFH) do not cross the placenta while low molecular weight
heparin do cross.
626- As regard warfarin therapy in thromboembolic diseases:
a) It crosses the placenta & results in effects on the fetus.
b) Breast feeding is a contraindication for this treatment.
c) It is usually started after complete stoppage of heparin treatment to avoid cumulative
anticoagulant doses of both drugs.
d) Its dose is controlled by measurements of partial thromboplastin time (PTT).
e) It is the best treatment during labor
627- In management of thromboembolic diseases, the following is true:
a) Thrombolytic therapy has a role in treatment of pulmonary thromboembolism (PTE).
b) Pulmonary embolectomy is contraindicated in critically ill pregnant lady with PTE.
c) Heparin SC is the 1st line of treatment for PTE.
d) Heparin in a dose of 10.000 unit IV dissolve already formed thrombus from
pulmonary vasculature in a patient with PTE.
e) Platelets must be evaluated with heparin therapy that may increase their count.
628- Which of the following is NOT likely in a patient with acute pulmonary embolism?
a) Tachycardia.
b) Tachypnea.
c) Increased alveolar-arterial gradient.
d) PaO2 >80 mm Hg.
e) Pleuritic chest pain.
629- Rh negative mother married to Rh positive father, the following is the
INCORRECT statement:
a) First baby usually passed unaffected.
b) Anti D immunoglobulin must be taken after abortion, ectopic vesicular mole,
chorionic villus sampling.
c) Anti D immunoglobulin is best given within the first week after delivery.
d) She may get Rh negative baby.
e) She may be sensitized during her first pregnancy due to weak placental barrier.

630- Which of the following maternal antibodies does NOT cause erythroblastosis
fetalis in the fetus?
a) Anti-C.
b) Anti-E.
c) Anti-D.
d) Anti-Lewis.

631- The incidence of Rh isoimmunization after a full-term delivery in a D-positive


infant to an Rh-negative mother when no Rh immune globulin prophylaxis has
been undertaken is:
a) 1%.
b) 5%.
c) 16%.
d) 40%.
e) 90%.

632- Which of the following characteristics and complications of pregnancy does NOT
in crease the risk of Rh sensitization?
a) Type of delivery.
b) Vaginal bleeding.
c) Preeclampsia.
d) Maternal age.

633- Possible episodes causing sensitization of Rh negative mother include all the
following EXCEPT:
a) Spontaneous or elective abortion.
b) Previous transfusion with Rh negative blood.
c) Previous delivery of Rh positive fetus.
d) Amniocentesis.
e) Ectopic pregnancy.

634- Additional increase in the risk of immunization in RH negative mother occurs in


all the following EXCEPT:
a) Cesarean section.
b) Traumatic delivery.
c) Placenta abruption.
d) Manual removal of placenta.
e) During labor.

635- In RH sensitized mother the percutaneous umbilical cord blood sampling (PUBS)
used to detect all the following EXCEPT:
a) Level of antibodies using indirect comb test.
b) Fetal blood group and RH.
c) Fetal HB and HCT level.
d) Bilirubin level.
e) Reticulocytic count.

636- RH immune globulin is given to unsensitized woman as prophylaxis as follows


EXCEPT:
a) At 20 weeks gestation.
b) Postpartum within 72 hours.
c) After spontaneous abortion or ectopic.
d) 300 ug after full term delivery.
e) 100 ug if pregnancy terminated before 13 weeks.

Directions Questions 637 through 650: Each set of matching questions in this section
consists of a list of 4 to 8 lettered options followed by several numbered items. For each
item, select the ONE best lettered option that is most closely associated with it. Each
lettered heading may be selected once, more than once, or not at all.

Questions 637-643
Match the antihypertensive drug with the best effect or the side effect mentioned
a) Nifedipine
b) Methyldopa
c) Atenolol
d) Hydralazine
e) Captopril

637- acts on the a receptors in the brain stem


638- causes tachyphylaxis if given orally
639- produces fetal renal failure
640- drug of choice in acute severe hypertension
641- causes lupus-like syndrome
642- is given as tocolytic as well
643- may cause fetal heart block

Questions 44- 650


Match the class of diabetes mellitus with pregnancy with the clinical picture given
below

a) Class A
b) Class B
c) Class C
d) Class D
e) Class R
f) Non of the above
644- Pregnant woman who is 21 years old and had known diabetes for 12 years
645- Pregnant woman who is 35 years old and had known diabetes for 5 years
646- Pregnant woman who is 21 years old and had developed diabetes at 29 weeks
gestation
647- Pregnant woman who is 18 years old and had known diabetes for 3 years
648- Pregnant woman who is 21 years old and had renal transplantation
649- Pregnant woman who is 23 years old and had proliferative retinopathy
650- Pregnant woman who is 27 years old and had nephropathy

Directions: Each of the following numbered items or incomplete statements in this


section is followed by numbered items that in certain combination leads to the BEST
lettered combined answer or/and completion. According to the following directions,
choose the BEST lettered answer:
If
1,2,3 are correct, choose A
1,3 are correct, choose B
2,4 are correct, choose C
1,2,3,4 are correct, choose D
AH are correct, choose E

651- The pathophysiology of preeclampsia is characterized by:


1- Vasospasm.
2- Hypervolemia.
3- Thromboxane is increased.
4- Hemodilution.
5- Vasodilatation followed by vasospasm.

652- A diagnosis of severe pre-eclampsia in a patient at thirty-seven weeks gestation


with a blood pressure of 160/110 mmHg would be supported by:
1 - Proteinuria of nephrotic range.
2- Ankle swelling.
3- Epigastric pain.
4- A blood pressure at 8 weeks gestation of 160/110 mmHg.
5- A parity of four.

653- Eclampsia:
1- Causes respiratory failure.
2- Causes hyperpyrexia.
3- Causes oliguria.
4- treated by intravenous diuretics.
5- Occurs more commonly at high altitude.
654- Tn the presence of bacterial endocarditis or its sequelae which of the following
statements are correct?
1- Antibiotic prophylaxis is necessary for all women in labor who have had
previous bacterial endocarditis.
2- Antibiotic prophylaxis is necessary for all women in labor who have undergone
previous reconstructive cardiac surgery.
3- Antibiotic prophylaxis is necessary for all women in labor with prosthetic heart
valves.
4- In obstetric practice bacterial endocarditis is usually caused by fecal streptococci.
5- A single intramuscular injection of 80 mg gentamicin at the onset of labor would
provide adequate antibacterial cover.

655- WHO defines anemia with pregnancy as follow:


1 - Hemoglobin level below 11 g/dl.
2- MCV value below 80 fl.
3- Hematocrit value less than 33%.
4- MCHC below 33%.
5- Reticulocytic count less than 2%.

656- Management of iron deficiency anemia with pregnancy include the following:
1 - The recommended oral dose of elemental iron is 150 •- 300 mg/day.
2- Oral therapy raises the Hb level 1 g/week.
3- Reticulocytic count begins to rise on starting iron therapy.
4- Main complication of parenteral iron therapy is its teratogenic effect.
5- Adding calcium preparation to iron formula is better than iron formula alone.

657- Anemia in pregnancy may increase in:


1 - Iron deficiency.
2- Chronic pyelonephritis.
3- Folic acid deficiency.
4- Multiple pregnancy.
5- Vitamin D deficiency.

658- Which of following substances are essential for normal erythropoiesis:


1- Cyanocobalamin.
2- Pyridoxine.
3- Folic acid.
4- Iodine.
5- Zinc.

659- Physiological changes with pregnancy include the following:


1 - Dilatation of the renal pelves.
2- Increase in the size of the kidneys.
3- Increased frequency of micturition.
4- Decrease of the renal plasma flow.
5- More marked compression of the left than the right ureter.
660- Ureteral dilatation during pregnancy is caused by which of the following:
1 - Uterine pressure at the pelvic brim.
2- Pressure from the presenting part.
3- Progesterone effect.
4- Pressure of polyhydramnios.
5- The increased glomerular filtration rate.
661- The following applies in patients with gestational diabetes mellitus:
1 - Gestational diabetes mellitus disappears after delivery.
2- Insulin is not essential in the management.
3- There is liability to hypoglycemia during labor.
4- Lactation is contraindicated after delivery.
5- Ventricular septal defect is a characteristic feature of congenital fetal anomalies
associated with diabetic pregnancy.
662- Obstetric complications of diabetes mellitus include the following:
1- Congenital fetal malformations.
2- Intrauterine fetal death.
3- Macrosomia.
4- Caput succedaneum.
5- Oligohydramnios.
663- Neonatal complications of infants of diabetic mothers includes the following:
1- Neonatal hypoglycemia.
2- RDS.
3- Hypocalcemia.
4- Hyperbilirubinemia.
5- Birth trauma.
664- In diabetic pregnancy
1- Congenital abnormalities are commoner
2- Insulin requirements increase
3- Monilial vaginitis is common
4- Intrauterine growth retardation is common
5- Oligohydramnios occurs
665- The severity of fetal affection in Rh sensitized woman can be detected by:
1- Ultrasound.
2- Detailed history.
3- Amniocentesis.
4- Percutaneous umbilical cord blood sampling (PUBS).
5- Antibodies screening using indirect Coombs test.

666- In a patient who is 12 weeks pregnant, severe atypicalities in a cervical smear


indicate a need for:
1- Colposcopy.
2- Random punch biopsy of the cervix.
3- A repeat cervical smear.
4- Termination of pregnancy.
5- Cone biopsy.

667- Fibroid with pregnancy:


1 - Is the commonest gynecological tumor with pregnancy.
2- It increases risk of abortion and PTL.
3- Associated with malpresentation.
4- May associated with post partum hemorrhage.
5- Commonly complicated with red degeneration in pregnancy.

668- Fibroid with pregnancy:


1 - May predispose to retained placenta and postpartum hemorrhage.
2- May predispose to abortion and PTL.
3- The tumor increases in size during pregnancy.
4- Red and cystic degeneration may occur during pregnancy.
5- Myomectomy is preferably done during cesarean section.

669- Ovarian tumor with pregnancy:


1 - Most of them are functional.
2- Most of them disappear by 14 weeks.
3- Teratoma accounts for 25% of ovarian tumors with pregnancy.
4- Serous cystadenoma more common than mutinous.
5- Most ovarian swellings with pregnancy are more than 5 cm.

670- Cancer cervix with pregnancy:


1 - Pregnancy does not affect the growth of carcinoma in situ.
2- The commonest presentation in pregnancy is bleeding.
3- Cone biopsy in pregnancy is associated with increased risk of abortion and PTL.
4- If detected before 30 weeks gestation hysterectomy is done followed by external
radiation.
5- If detected after 30 weeks gestation treatment is delayed till 35 weeks.

671- Retroverted gravid uterus:


1 - Spontaneous correction usually occurs by 12 weeks.
2- Accounts for 20% of all gravid uteri.
3- Impacted retroverted gravid uterus may causes acute abdomen.
4- Impacted retroverted gravid uterus may causes abortion.
5- Impacted retroverted gravid is common in contracted pelvis.

672- Causes of acute abdomen with pregnancy include:


1 - Placenta abruption.
2- Complicated fibroid.
3- Ruptured tubal pregnancy.
4- Complicated ovarian cyst.
5- Placenta previa.

673- Causes of acute abdomen with pregnancy include:


1 - Acute pyelonephritis.
2- Acute appendicitis.
3- Complicated fibroid.
4- Sickle cell crisis.
5- Chorioamnionitis.
674- Acute appendicitis with pregnancy:
1 - Is the most common surgical complication with pregnancy.
2- Incidence about 1/500.
3- Differential diagnosis includes ectopic pregnancy and accidental hemorrhage.
4- Laparotomy is indicated regardless gestational age.
5- Right lower quadrant pain and rebound tender usually present.

Directions: In the following questions, each item has 5 lettered options each of which
could be a true or a false statement. Tick each of these options according to whether
they are false or true

675- As normal pregnancy progresses the following hematological changes may occur:
a) Plasma volume increases and red cell volume remains constant.
b) Plasma volume increase proportionately more than the increase in red cell volume.
c) Red cell volume decreases and plasma volume remains constant.
d) Neither plasma volume nor red cell volume change.
e) This hematological changes ordinarily occurring during normal pregnancy may
result in anemia (physiological anemia).

676- In management of iron deficiency anemia:


a) Oral iron therapy usually lead to diarrhea.
b) Hematological response to intramuscular iron therapy is faster than that of oral
therapy.
c) 300 mg of iron taken orally each day is adequate.
d) Total dose infusion of iron is contraindicated.
e) Packed RBCs transfusion results in rapid correction of the anemia (emergency
treatment).

677- Anemia in pregnancy:


a) Is defined as hemoglobin of 10 gm% or less.
b) May be caused by hemodilution of pregnancy.
c) May be caused by hook worm infestation.
d) When megaloblastic is usually due to B12 deficiency.
e) Is relatively common in multiple pregnancy.

678- Asymptomatic bacteriuria in early pregnancy:


a) Is found in 12% of pregnant women.
b) Is only regarded as significant when the bacterial count is more than 105
organisms/ml.
c) Leads in later pregnancy to pyelonephritis in 25% of cases.
d) Should be treated with antimicrobial agents.
e) Is usually caused by streptococcus fecalis.

679- Pregnancies complicated by poorly controlled insulin dependent diabetes mellitus


are complicated by:
a) Delayed fetal pulmonary maturation.
b) Decreased amniotic fluid magnesium concentration.
c) Progression of retinopathy faster than in well controlled diabetes.
d) Neonatal hypomagnesemia
e) Neonatal hyperglycemia.
680- Insulin adjustment during pregnancy:
a) Make diet manipulation unnecessary.
b) Three-injection regimen is needed in most cases.
c) The evening dose is the usually higher than the others.
d) Timing of injection is about 20-30 minutes before meals.
e) Portable glucose meters are useful.
681- Gestational diabetes mellitus:
a) Should be screened for in all pregnancies.
b) May be excluded by a fasting glucose.
c) Oral glucose tolerance test is reliably reproducible.
d) Is an indication for induction at 38 weeks.
e) Is a precursor of diabetes in later life.
682- Diabetes in pregnancy:
a) Diabetes may develop owing to the anti-insulin effects of pregnancy-related
hormones such as HPL and cortisol.
b) Maternal glucose crosses the placenta into the fetal circulation via active transport
mechanisms.
c) A glycosylated HbA1c of greater than 6% is diagnostic of diabetes.
d) May be controlled by oral hypoglycemic drugs during the first trimester.
e) Insulin requirement may be decreased during the first trimester.
683- Pregnancy has which of the following effects on diabetic women:
a) Tendency toward ketosis during early pregnancy.
b) Tendency toward hyperglycemia during early pregnancy.
c) Increase in insulin requirement during early pregnancy.
d) Increase in insulin requirement during late pregnancy.
e) Women, with preexisting but undetected overt DM, may be initially diagnosed
during pregnancy.
684- The following contribute to good control of diabetes in pregnancy:
a) 400 grams per day carbohydrate diet.
b) Carbohydrates accounts for 50% of total energy intake.
c) Oral hypoglycemic drugs in mild cases.
d) Twice-daily mixtures of short and medium acting insulin.
e) Elimination of infection.
685- Treatment of thyrotoxicosis during pregnancy:
a) Propylthiouracil and methimazole are principle medication used.
b) Subtotal thyroidectomy is usually indicated.
c) Radioactive iodine could be used in certain cases.
d) Aim of beta-blocking agents is to maintain lowest possible doses of antithyroid
agents.
e) Agranulocytosis is the most serious complication of antithyroid drugs.
686- Effects of hypothyroidism on pregnancy include:
a) Increased risk of abortion.
b) Fetal macrosomia.
c) Cretinism may occur.
d) Gestational diabetes mellitus.
e) Heart failure.

687- As regard amniotic fluid embolism (AFE):


a) AFE is more serious fatal than PTE (pulmonary thromboembolism),
b) AFE occurs more often after rupture of membranes than in a patient with intact
membranes.
c) Autopsy is of no value in retrospective diagnosis of AFE.
d) Pregnancy induces a hypercoagulable blood state.
e) The commonest cause is premature rupture of membranes.

688- In disseminated intravascular coagulopathy (DIG):


a) Fibrin degradation products (FDP) are increased in cases with consumption
coagulopathy.
b) Heparin should never be given in a case with DIG.
c) Clinical evidence of bleeding with the presence of a predisposing factor is often the
1st sign of occurrence of DIG.
d) DIG should be anticipated & blood tests should be performed for early diagnosis.
e) Excessive blood transfusion may be a cause.

689- Acute appendicitis with pregnancy:


a) Most common surgical complication with pregnancy.
b) Classical signs are often absent.
c) Antibiotics are usually indicated.
d) May be complicated by PTL.
e) Maternal mortality may occur in 50% because of surgical delay.
ABNORMAL LABOR

Directions: Each of the numbered items or incomplete statements in this section is


followed by answers or by completions of the statement. Select the ONE lettered answer
or completion that is BEST in each case.

690- The following statements regarding breech presentation are correct EXCEPT:
a) Uterine septum can cause breech presentation.
b) The commonest cause is hydrocephalus.
c) The commonest cause of neonatal death after vaginal delivery is intracranial
hemorrhage.
d) A safe method of delivery of the after-coming head is by forceps.
e) Prolapse of the cord has good prognosis with breech presentation compared to
cephalic presentation.

691- The following conditions may predispose to a breech presentation EXCEPT:


a) Hydrocephalus.
b) Contracted pelvis.
c) Placenta previa.
d) Prematurity
e) Septate uterus.

692- A complete breech presentation is best described by which of the following


statements denoting fetal parts:
a) The legs and thighs are flexed.
b) The legs are extended, and the thighs are flexed.
c) The arms, legs, and thighs are flexed completely.
d) The legs and thighs are extended.
e) The legs and thighs are extended but back is flexed.

693- The following statements regarding face presentation are correct EXCEPT:
a) It occurs once in 300 cases.
b) The dominator is the chin.
c) The engaging diameter equals the Suboccipito bregmatic diameter in length.
d) Diagnosis is difficult by abdominal palpation.
e) The commonest cause is hydrocephalus.

694- In twin pregnancy, the following are true EXCEPT:


a) Monozygotic twins usually have a single placenta.
b) Dizygotic twins have a familial trait.
c) Monozygotic twin rates are influenced by paternal factors.
d) The incidence of monozygous twins is 0.4-0.5%.
e) Proteinuric pregnancy-induced hypertension is more common in dizygotic twins.

695- As regard the etiology of twin pregnancy, the following is correct:


a) Diamniotic dichorionic twins occurs if division occurs after chronic differentiation.
b) Cleavage of the embryo after amnion differentiation result in DA MC twinning.
c) Conjoined twins results from early cleavage of embryo in first 3 days of conception.
d) Maternal positive family history is more important than paternal side.
e) Superfetation could occur in human, while superfecundation never occur.
696- In management of multifetal pregnancy, the following is true:
a) Extra antenatal care is needed for a pregnant lady with multi-fetal pregnancy.
b) In multi-fetal pregnancies, the smaller the fetal weight (<2kg) the more likely the
vaginal route is chosen.
c) In multi-fetal pregnancies the larger the fetal number, the more the chance of vaginal
delivery due to smaller size.
d) Mono-amniotic twins are associated with easier vaginal delivery & less perinatal
mortality than diamniotic twins.
e) Internal podalic for the 1st twin is possible as it may have smaller size compared to a
singleton.

697- The following statements regarding twin pregnancy are correct EXCEPT:
a) Cephalic-cephalic twin presentation is the most common presentation.
b) Incidence of twinning as diagnosed by early US is similar to its incidence at birth.
c) Multifetal pregnancy could be suspected if a pregnant lady had an early threatened
abortion.
d) Extra antenatal care is needed for a pregnant lady with multifetal pregnancy.
e) Examination of the placenta is important determination of fetal zygosity.

698- Complications of polyhydramnios include the following EXCEPT:


a) Preterm labor.
b) Preterm premature rupture of membranes.
c) Antepartum hemorrhage.
d) Postpartum hemorrhage.
e) Postmaturity.

699- Oligohydramnios may be associated with the following EXCEPT:


a) Postdate pregnancy.
b) Chronic placental insufficiency.
c) esophageal atresia.
d) Renal agenesis in the fetus.
e) Rupture of membranes.
700- In the diagnosis of premature rupture of membranes:
a) Speculum examination is rarely done.
b) History of sudden gush of water from the vagina is a sure symptom of
premature rupture of membranes.
e) Nitrazine paper turns deep blue.
d) Positive fern test is a sure test for diagnosis.
e) C-reactive protein is a specific indicator for chorioamnionitis.

701- Termination of pregnancy in cases of preterm premature rupture of membranes


occur in the following conditions EXCEPT if:
a) The patient goes in active labor.
b) There are uterine contractions 1/15 minutes.
c) Fetal distress occurred.
d) Presence of fever after excluding extrauterine causes of fever.
e) Cervix is 5 cm dilated.

702- A healthy primigravida is admitted at 30 weeks' gestation with spontaneous


rupture of the membranes. The following statement is correct:
a) Fetal pulmonary hypoplasia may occur due to oligohydramnios.
b) Prophylactic antibiotics are contraindicated.
c) Corticosteroids are contraindicated because of the risk of infection.
d) Active management results in a better perinatal outcome than is achieved by
expectant management.
e) The risk of preterm prelabor rupture of the membranes in subsequent pregnancies is
20%.

703- Naegele's pelvis is:


a) Arrested development of both ala of sacrum.
b) Arrested development of one ala of the sacrum.
c) Arrested development of the sacrum.
d) Separation of both pubic bones.
e) Absence of the coccyx.

704- Fetal complications of contracted pelvis during labor include the following
EXCEPT:
a) Caput succedaneum.
b) Over molding.
c) Intra cranial hemorrhage.
d) Fetal malformation.
e) Fetal death.

705- Contra indication of trial of labor in patient with contracted pelvis include the
following EXCEPT:
a) Non vertex presentation.
b) Mild degree of cephalopelvic disproportion.
c) Previous C.S.
d) Elderly primigravida.
e) Heart disease with pregnancy.

706- The following finding are suggestive of contracted pelvis during examination
EXCEPT:
a) Kyphosis.
b) Non-engagement of the head.
c) Limping gait.
d) Height 170cm.
e) Pendulous abdomen in primigravida.

707- Precipitate labor & delivery may lead to the following complications EXCEPT:
a) Uterine rupture.
b) Amniotic fluid embolism.
c) Vagina & perineal tears.
d) Constriction ring in the uterus.
e) Uterine atony.

708- Uterine atony after cesarean section is treated by the following EXCEPT:
a) Uterine massage.
b) Intramyometrial syntocinon injection.
c) Intravenous infusion of methergine.
d) Intramyometrial injection of prostaglandin F2α.
e) Bilateral hypogastric artery ligation.
709- Management of shoulder dystocia may include the following procedures EXCEPT:
a) Vigorous fundal pressure.
b) Corkscrew rotation of the shoulders.
c) Generous episiotomy.
d) Shoulder girdle rotation into one of the oblique diameters of the pelvis.
e) Attendance of expert neonatologist.

710- The following statements regarding rupture of the uterus are correct EXCEPT:
a) More common in multiparous than primiparous women.
b) Its complete type is more common in the upper uterine segment.
c) In its incomplete type, the uterine cavity is continuous with the peritoneal cavity.
d) Can occur in primiparous women due to inappropriate use of oxytocin during labor.
e) Its incidence varies according to the standard of antenatal care.

711- The following statements regarding cesarean section (CS) scar rupture are correct
EXCEPT:
a) Upper segment CS scar rupture 2-4%.
b) This is due to improper coaptation of the edges in USCS scar.
c) Upper segment scar is overstretched by subsequent pregnancies.
d) Upper segment of the uterus is passive during labor.
e) Infection is more common in the upper segment CS.

712- The following statements regarding rupture of the uterus are correct EXCEPT:
a) It may occur during manual separation of the placenta.
b) During labor, may occur due to extension of old cervical tear.
c) During pregnancy, is almost always due to a traumatic cause.
d) During pregnancy, presents with antepartum hemorrhage.
e) It can occur due to Couvelaire uterus.
713- Uterine rupture is more common in multiparous women due to increased incidence
of all the following EXCEPT:
a) malpresentation.
b) intra-uterine growth restricted fetus.
c) pendulous abdomen.
d) osteomalacia.
e) uterine wall weakness.
714- Incomplete uterine rupture is characterized by:
a) Massive hemorrhage.
b) All layers of the uterus are involved.
c) The fetus is extruded outside the uterus.
d) The visceral peritoneum is intact.
e) The fetus is almost always dead.
715- Which of the following is NOT a complication of uterine rupture:
a) Maternal shock.
b) Renal failure.
c) Acute uterine inversion.
d) Paralytic ileus.
e) Fetal loss.
716- Uterine rupture is more common after upper segment cesarean section (USCS)
than lower segment one (LSCS) due to the following EXCEPT:
a) Improper coaptation of the edges.
b) The upper segment is much thicker than the lower.
c) Active upper segment during labor.
d) Infection is more common in the lower segment.
e) Placental implantation is more common at the upper segment.
717- All of the following are lines of management of uterine rupture in a patient 38
years old EXCEPT:
a) Total abdominal hysterectomy plus bilateral salpingo-oophorectomy (TAH & BSO)
b) Antishock measures.
c) Repair of the ruptured site.
d) Internal iliac artery ligation.
e) Angiographic directed arterial embolization.
718- In rupture of the uterus, the following may occur EXCEPT:
a) Hypotension.
b) Increased uterine contractions.
c) Hematuria.
d) Vaginal bleeding.
e) Fetal distress.
719- Manual removal of the placenta:
a) Is performed using a pudendal block as analgesia.
a) Has been superseded by the use of the suction curette.
b) Usually done by piece-meal extraction of placenta.
c) Should be performed if placenta failed to separate within 10 minutes.
d) Is an indication for giving prophylactic antibiotics.

720- The following statements regarding secondary postpartum hemorrhage are correct
EXCEPT:
a) It is defined as loss of more than 500 ml blood from the genital tract between 24
hours & 6 weeks after delivery.
b) It is usually due to retained products of the placenta.
c) It is frequently associated with uterine infection.
d) It commonly requires blood transfusion.
e) Choriocarcinoma is suspected if bleeding is recurrent.

Directions Questions 721 through 725: Each set of matching questions in this section
consists of a list of 4 to 8 lettered options followed by several numbered items. For each
item, select the ONE best lettered option that is most closely associated with it. Each
lettered heading may be selected once, more than once, or not at all.

Match the position or the presentation below with its approximate incidence:
a) 0.003%
b) 0.03%
c) 0.3%
d) 3.5%.
e) 20%

721- Shoulder presentation


722- Occipitoposterior position
723- Breech presentation
724- Face presentation
725- Brow presentation

Directions: Each of the following numbered items or incomplete statements in this


section is followed by numbered items that in certain combination leads to the BEST
lettered combined answer or/and completion. According to the following directions,
choose the BEST lettered answer:
If
1,2,3 are correct, choose A
1,3 are correct, choose B
2,4 are correct, choose C
1,2,3,4 are correct, choose D
All are correct, choose E
726- The following statements regarding occipito posterior position:
1- During delivery of direct occipito posterior, the perineum is distended by the
occipitofrontal diameter.
2- Deep transverse arrest is best managed by vacuum extraction.
3- The progress of labor depends on the degree of head flexion.
4- Android pelvis is a rare cause.
5- During labor, 1/3 of the cases deliver spontaneously as occipito anterior.

727- In breech presentation:


1- The commonest type is complete breech.
2- The commonest cause is prematurity.
3- The safest method of delivery of the aftercoming head of the breech is by
forceps.
4- The commonest cause of fetal death after vaginal delivery is ruptured liver.
5- Prolapse of the cord is more common with breech presentation compared to
transverse lie.

728- For twin infants to be monozygotic, which of the following characteristics must be
present:
1- Identical sex.
2- A single amniotic sac.
3- Identical major blood groups.
4- A single placenta
5- Absence of chorion between the two amnions.

729- The following are congenital fetal malformations causing polyhydramnios:


1- Duodenal atresia.
2- Anencephaly.
3- Omphalocele.
4- Renal agenesis.
5- Syndactyly.

730- Preterm premature rupture of membranes is rupture of membranes:


1- before reaching maturity.
2- at 36 weeks.
3- at 29 weeks.
4- at 39 weeks gestation.
5- at full term pregnancy.

731- Causes of contracted pelvis include the following:


1- Rickets.
2- Osteomalacia
3- Fractures of the pelvis.
4- T.B arthritis of the pelvic joints.
5- Osteomyelitis.
732- Which of the following disease or deformity of the lower limbs affects the pelvic
capacity:
1- Poliomyelitis.
2- Hip joint disease.
3- Fracture femur.
4- Varicose veins.
5- Leg ulcer.
734- Pathological retraction ring:
1- It is a complication of obstructed labor.
2- It is also called Bandl ring.
3- It can be felt and seen abdominally.
4- It occurs in the middle of the upper uterine segment.
5- It has a fixed position and does not rise up with labor progression.
735- Incoordinate uterine contractions includes:
1- Contraction ring.
2- Cervical dystocia.
3- Colicky uterus.
4- Retraction ring.
5- Cervical incompetence.
736- Primary postpartum hemorrhage is associated with:
1 - Placenta previa.
2- Polyhydramnios.
3- Forceps delivery.
4- The use of tocolytic agents.
5- Prolonged labor.
737- Treatment of ruptured uterus include the following:
1 - In a young patient with clean cut edges and no children, repair of tear + bilateral
internal iliac artery ligation may be considered.
2- Prophylactic treatment by good antenatal care is important.
3- Supravaginal hysterectomy + bilateral internal iliac artery ligation.
4- Immediate laparotomy through a Pfannenstiel incision.
5- Antishock measures are considered of no importance in comparison to
hysterectomy.
738- In the absence of an obvious vaginal hemorrhage, postpartum collapse may be due
to:
1- A ruptured uterus.
2- Uterine inversion.
3- Amniotic fluid embolism.
4- A paravaginal hematoma.
5- Eclampsia.
739- Amniotic fluid embolism:
1 - Causes cyanosis.
2- Commonly caused by accidental hemorrhage.
3- Is complicated by disseminated intravascular coagulopathy.
4- Is a complication of the puerperium.
5- Commonly caused by placenta previa.

740- Coagulation failure is an important complication of:


1 - Amniotic fluid embolus.
2- Gram-negative septicemia.
3- Abruptio placenta.
4- Placenta previa.
5- Ruptured ovarian cyst with pregnancy.

Directions: In the following questions, each item has 5 lettered options each of which
could be a true or a false statement. Tick each of these options according to whether
they are false or true

741- In multiple pregnancy:


a) The incidence of spontaneous twin pregnancy is 1:80.
b) Perinatal mortality is increased in triplet pregnancy by a factor often.
c) Intrauterine growth restriction is common and is the main etiological factor in the
increased perinatal mortality rate.
d) Some obstetricians recommended prophylactic cervical cerclage in triplet pregnancy.
e) The outcome of triplet is primarily related to fetal weight at delivery.

742- Obstructed labor:


a) Commonly occurs in cardiac patients.
b) Commonly occurs in primigravidas.
c) Ts reduced by proper maternal care.
d) Vaginal examination shows dry, warm vagina.
e) Best management is by instrumental delivery.

743- Clinical picture of ruptured uterus following obstructed labor:


a) Feeling of obstruction is replaced by feeling of giving way.
b) The patient becomes noisy and irritable.
c) PV examination reveals receding freely mobile presenting part.
d) Strong uterine contractions suddenly stop.
e) General condition of the patient is good.

744- Immediate therapy for a completely inverted uterus after delivery includes:
a) Maintaining sufficient intravenous lines.
b) Infusing oxytocin.
c) Administering nitroglycerine.
d) Emergency laparotomy.
e) Emergency vaginal hysterectomy.

745- Primary postpartum hemorrhage:


a) Can occur at any time in the first week after delivery.
b) Frequently occurs in association with the 3rd stage of labor.
c) Commonly due to a coagulation failure.
d) Is due to complete failure of separation of the placenta.
e) Is commonly due to uterine inertia.

746- Primary postpartum hemorrhage occurs more commonly:


a) In primigravidae.
b) When there is a history of a previous postpartum hemorrhage.
c) When fibroids are present in the uterus.
d) In premature labor.
e) After placental abruption.

748- Placenta percreta:


a) Is associated with placenta previa.
b) Should be managed by removal of as much placenta as possible.
c) Is associated with a previous cesarean section.
d) Necessitates hysterectomy.
e) Commonly involves invasion through to the serosal coat of the uterus.

749- Acute uterine inversion:


a) Only occurs with a relaxed uterus.
b) Is usually caused by applying fundal pressure.
c) Is managed by immediate removal of placenta before reposition of the uterus.
d) Can be managed by increasing the hydrostatic pressure in the vagina.
e) Requires hysterectomy.
THE NEWBORN
Directions: Each of the numbered items or incomplete statements in this section is
followed by answers or by completions of the statement. Select the ONE lettered answer
or completion that is BEST in each case.

750- Shortly after vaginal delivery of an infant in an occiput posterior position, a


vaguely demarcated edematous area over the midline of the skull was noted. This
observed lesion is most likely:
a) A cephalhematoma.
b) A skull fracture.
c) Caput succedaneum.
d) A subdural hematoma.
e) Subgaleal hematoma.

752- Causes of neonatal jaundice include the following EXCEPT:


a) Polycythemia.
b) Beta thalassemia.
c) Congenital biliary atresia.
d) Toxoplasmosis.
e) Excessive oxytocin administration during labor.

Directions: Each of the following numbered items or incomplete statements in this


section is followed by numbered items that in certain combination leads to the BEST
lettered combined answer or/and completion. According to the following directions,
choose the BEST lettered answer:
If
1,2,3 are correct, choose A
1,3 are correct, choose B
2,4 are correct, choose C
1,2,3,4 are correct, choose D
All are correct, choose E

753- The following statements regarding physiological neonatal jaundice are correct:
1- Almost 50% of all newborns have visible jaundice in the first week of life.
2- Appears in the first day after delivery.
3- Neonatal hypothyroidism may prolong the disappearance of jaundice.
4- Conjugated bilirubin increases more than unconjugated.
5- Total bilirubin usually exceeds 20mg/dl.

755- The following statements regarding caput succedaneum are true:


1 - Maximum size appears at birth & disappears after 24 hours.
2- Radiography may be recommended.
3- Follows prolonged and difficult labor.
4- The swelling does not cross suture lines.
5- Forceps delivery is the predisposing factor.

Directions: In the following questions, each item has 5 lettered options each of which
could be a true or a false statement. Tick each of these options according to whether
they are false or true

756- Cephalhematoma
a) Is hemorrhage under the periosteum of skull bone.
b) Instrumental delivery is the main predisposing factor.
c) The swelling disappeared within 24H after delivery.
d) Skull fractures are seen in approximately 60% of cases.
e) Anemia and hyperbilirubinemia are common sequelae in large cephalhematoma.

758- Respiratory distress syndrome


a) Is common in neonates of pre-ecliptic mothers.
b) Corticosteroid before termination of pregnancy has a preventive role.
c) In diabetic patient, detection of phosphatidyl glycerol in amniotic fluid is the most
specific & sensitive test in detection of lung maturity.
d) Management of RDS may be complicated by retrolental fibroplasia.
e) RDS starts to present from second day after delivery.
OPERATIVE OBSTETRICS

Directions: Each of the numbered items or incomplete statements in this section is


followed by answers or by completions of the statement. Select the ONE lettered answer
or completion that is BEST in each case.

759- Which of the following is NOT a characteristic of a mediolateral episiotomy?


a) Difficulty of repair.
b) Common faulty healing.
c) Occasional dyspareunia.
d) Relatively common extension through anal sphincter and into rectum.
e) Occasional faulty anatomic result.

760- Advantage of a median episiotomy include the following EXCEPT:


a) Increased area of vaginal outlet to facilitate delivery.
b) Less blood loss compared to mediolateral technique.
c) Avoidance of major perineal lacerations.
d) Decreased risk of injury to the anal sphincter and mucosa.
e) Greater ease of repair compared to mediolateral technique.

761- The following are associated with a successful external cephalic version EXCEPT:
a) Frank breech.
b) Large amniotic volume.
c) Unengaged fetus.
d) High parity.
e) Use of tocolytics.

762- The following statements regarding instrumental vaginal delivery are correct
EXCEPT:
a) It is common in laboring women with epidural anesthesia.
b) With the vacuum extractor, it is too slow to be useful when rapid delivery is
required.
c) It may be avoided by the appropriate use of syntocinon in the second stage of labor.
d) It leads to third degree perineal tears more frequently with forceps than with the
vacuum extractor.
e) The vacuum extractor is associated with more maternal trauma than the obstetric
forceps.

763- Causes for increased CS rate include the following EXCEPT:


a) Dystocia.
b) Breech presentation.
c) Fetal distress.
d) Intra-uterine fetal death.
e) Repeat CS.

764- Regarding preoperative preparations for CS, the following statements are true
EXCEPT:
a) The bladder must be full to identify its edge during the operation.
b) Adequate counseling and consent is essential.
c) The wound site is prepared with antibacterial wash.
d) Blood is prepared for high risk pregnancies.
e) Anesthetic evaluation is ordered.

765- Regarding anesthesia for CS, the following statements are true EXCEPT:
a) Local anesthesia is used in certain conditions.
b) Maternal position during the procedure is very important.
c) Anesthetic complications are important causes for maternal mortality.
d) Postoperative aspiration is a common cause of maternal mortality.
e) Epidural anesthesia is associated with postoperative aspiration.

766- The following statements regarding cesarean section are true EXCEPT:
a) It is safer than vaginal delivery for the preterm breech.
b) Associated infection is markedly reduced by the use of prophylactic antibiotics.
c) Performed electively under regional anesthesia is as safe for the mother as normal
vaginal delivery.
d) It is commonly performed for prolonged labor (dystocia).
e) The lower segment one is safer than the upper segment one.

767- Indications of caesarean section include all the following EXCEPT:


a) Previous classic C.S.
b) Failed forceps delivery.
c) Non reassuring fetal heart rate tracing.
d) McDonald cervical cerclage.
e) Cord prolapse.

768- Criteria to allow vaginal birth after cesarean section (C.S.) include the following:
a) The previous C.S was because of contracted pelvis.
b) The previous C.S was a classic one (upper segment).
c) Oversized fetus in the current pregnancy.
d) Breech presentation in the current pregnancy.
e) The post partum period following the C.S was uneventful.

769- Advantages of lower segment C.S over upper segment C.S. include the following
EXCEPT:
a) Less bleeding unless extended.
b) Less paralytic ileus.
c) Used if there are pelvic adhesions.
d) Less incidence of subsequent uterine rupture.
e) Less infection.

Directions: Each of the following numbered items or incomplete statements in this


section is followed by numbered items that in certain combination leads to the BEST
lettered combined answer or/and completion. According to the following directions,
choose the BEST lettered answer:
If
1,2,3 are correct, choose A
1,3 are correct, choose B
2,4 are correct, choose C
1,2,3,4 are correct, choose D
All are correct, choose E
770- Indications of use of vacuum extractor in assisted vaginal delivery include:
1. Fetal distress in the second stage.
2. Borderline cephalopelvic disproportion.
3. Shortening the second stage in hypertensive mothers.
4. Suspected fetal coagulopathy.
5. Face presentation.

771- The obstetric forceps:


1. Should be used only to assist delivery of cephalically presenting fetus.
2. Should not be used vaginally unless the fetal head is engaged.
3. Should be used routinely to assist vaginal delivery of the preterm infant.
4. Can be used to assist delivery of the fetal head during C.S.
5. May be used in face presentation when the chin is directed to towards the sacrum
(mentoposterior).

772- Pre requisites for a forceps delivery include:


1. A completely dilated cervix.
2. Ruptured membranes.
3. Engagement of the head.
4. An empty bladder.
5. The occipito-anterior position.

773- Regarding traction with forceps:


1. Traction should be intermittent.
2. Episiotomy is essential.
3. Delivery should be slow.
4. It is preferable to apply traction between contractions to avoid excessive
pressure.
5. The fetal head should be allowed to recede in intervals.

774- Advantages of prophylactic episiotomy:


1. Less incidence of dyspareunia.
2. Reduction in duration of second stage of labor.
3. Decreased blood loss.
4. Reduction of subsequent pelvic relaxation.
5. Avoidance of damage to the rectal sphincter in all types of episiotomy.
775- Indications of C.S hysterectomy include the following:
1. Severe uterine atony.
2. Placenta accreta.
3. Irreparable uterine rupture.
4. Subserous leiomyoma.
5. Repeat C.S.

776- Indications of induction of labor include the following:


1. Pre-eclampsia.
2. PROM at 38 weeks gestation.
3. Fetal demise.
4. Post term pregnancy.
5. Placenta previa with mild bleeding, no labor pains at 30 weeks.

777- Side effects of oxytocin include:


1. Fetal distress.
2. Uterine rupture.
3. Hypotension.
4. Amniotic fluid embolism.
5. Hypernatremia.

778- Regarding postoperative care after CS:


1. Oral fluids 8 to 12 hrs can be started after surgery.
2. Observations for the vital data are essential.
3. Adequate pain medication is essential.
4. Early ambulation is encouraged.
5. Breathing & leg exercises are encouraged.

779- Indications of therapeutic abortion include:


1. Advanced hypertensive vascular disease.
2. Carcinoma of the cervix
3. Active pulmonary T.B.
4. All cases of uncontrolled D.M.
5. Breast carcinoma with pregnancy.

780- Complications of evacuation and curettage of the uterus include:


1. Uterine perforation.
2. Infection.
3. Cervical laceration.
4. Hemorrhage.
5. Asherman's syndrome.

781- Oxytocin:
1.

Directions: In the following questions, each item has 5 lettered options each of which
could be a true or a false statement. Tick each of these options according to whether
they are false or true

782- A classical cesarean section:


a) Should be done in cases of transverse lie.
b) Is performed when large fibroids occupy the lower segment of the uterus.
c) Is performed in most cases of placenta previa.
d) Is performed through a transverse incision in the upper segment of the uterus.
e) Scar is liable to rupture after 30 weeks gestation.

783- Regarding CS:


a) It is delivery of the infant through incisions in the abdominal and uterine wall after
viability.
b)
c) There are many absolute contraindication for CS:
d) CS is better avoided in fetal demise.
e) CS is better avoided in major malformations incompatible with life.

784- Oxytocin:
a) Has 50 amino acids.
b) Is synthesized in the posterior pituitary.
c) Is formed as larger precursor molecule.
d) Is released by the fetus during labor.
e) Is found in the corpus luteum.
Is a peptide hormone.
2. The uterine sensitivity to it increases towards the end of pregnancy.
3. Has some anti-diuretic action.
4. Is synthesized in the posterior lobe of the pituitary gland.
5. Secretion is stimulated by alcohol.
ANSWERS

GYNECOLOGY

1 C 10 B 19 D 28 A 37 E 46 E
2 D 11 E 20 B 29 E 38 C 47 A
3 A 12 A 21 B 30 D 39 D 48 A
4 E 13 C 22 C 31 C 40 B 49 B
5 D 14 C 23 A 32 E 41 E 50 A
6 A 15 B 24 D 33 D 42 G 51 A
7 E 16 A 25 A 34 B 43 F 52 B
8 A 17 C 26 D 35 C 44 H 53 C
9 D 18 C 27 B 36 A 45 A 54 A

a b c d e a b c d e
55 F T F T F 60 F F T T F
56 F F T T F 61 F T T F F
57 F T F F T 62 T T F F F
58 T F T F F 63 T F F T T
59 T F F F T 64 T F T F T

65 E 78 E 91 C 104 E 117 C 130 A


66 B 79 D 92 B 105 B 118 A 131 C
67 E 80 C 93 E 106 A 119 B 132 A
68 B 81 E 94 C 107 D 120 C 133 A
69 C 82 D 95 E 108 B 121 A 134 B
70 C 83 C 96 D 109 A 122 D 135 A
71 C 84 C 97 D 110 C 123 A 136 B
72 E 85 B 98 F 111 C 124 B 137 B
73 C 86 D 99 D 112 B 125 A 138 B
74 C 87 C 100 C 113 A 126 A 139 A
75 E 88 A 101 E 114 A 127 B
76 E 89 D 102 B 115 C 128 A
77 D 90 C 103 C 116 B 129 A

a b c d e a b c d e
140 T T F T F 142 F T F F T
141 T T F T F 143 F F T T F

144 A 148 B 152 E 156 A 160 A 164 A


145 C 149 E 153 F 157 B 161 B
146 E; 150 B 154 D 158 B 162 A
147 B 151 D 155 C 159 D 163 A

a b c d e a b c d e
165 F T T T F 167 F F T T T
166 T T T F F
168 C 175 B 182 B 189 B 196 A 203 D
169 D 176 B 183 A 190 A 197 H 204 C
170 C 177 D 184 B 191 G 198 G
171 C 178 C 185 C 192 C 199 F
172 A 179 B 186 B 193 G 200 B
173 C 180 B 187 B 194 E 201 E
174 B 181 A 188 A 195 C 202 D

a b c d e a b c d e
205 F F T T F 207 T F T T F
206 F T T T F

208 E 218 C 228 B 238 B 248 A 258 E


209 C 219 C 229 D 239 B 249 D 259 A
210 C 220 A 230 A 240 E 250 A 260 B
211 D 221 A 231 C 241 B 251 D 261 B
212 A 222 B 232 B 242 B 252 D 262 B
213 B 223 D 233 B 243 B 253 E
214 A 224 A 234 B 244 A 254 B
215 B 225 A 235 B 245 D 255 E
216 D 226 A 236 E 246 E 256 C
217 F 227 B 237 C 247 B 257 A

a b c d e a b c d e
263 F T T F F 265 T F T F T
264 T F T F F 266 T T T T F

267 A 285 A 303 B 321 A 339 C 357 A


268 C 286 A 304 A 322 B 340 E 358 C
269 D 287 B 305 B 323 B 341 D 359 E
270 C 288 D 306 A 324 B 342 A 360 E
271 C 289 B 307 B 325 E 343 C 361 B
272 D 290 B 308 D 326 C 344 F 362 B
273 B 291 E 309 A 327 C 345 II 363 E
274 A 292 A 310 C 328 D 346 F 364 A
275 E 293 A 311 C 329 F 347 D 365 A
276 E 294 B 312 B 330 G 348 B 366 B
277 C 295 B 313 D 331 B 349 B 367 A
278 A 296 C 314 A 332 A 350 A 368 D
279 C 297 D 315 C 333 A 351 B 369 A
280 B 298 D 316 D 334 A 352 D 370 D
281 A 299 C 317 B 335 D 353 B
282 D 300 B 318 D 336 B 354 B
283 C 301 C 319 B 337 C 355 B
284 D 302 D 320 C 338 B 356 B
a b c d e a b c d e
371 T F F T T 375 F T T T T
372 F T T F T 376 F F T T F
373 T F T T T 377 T F F F T
374 rT T T T F 378 T T F F T

379 E 383 C 387 A 391 F 395 B 399 A


380 B 384 D 388 E 392 A 396 B 400 B
381 D_ 385 D 389 F 393 A 397 A
382 C 386 D 390 E 394 E 398 A

a b c d e a b c d e
401 T T F F T 404 T F F T T
402 T F T T T 405 T F T T F
403 T T T T T

OBSTETRICS
406 E 419 E 432 B 445 B 458 A 471 A
407 B 420 B 433 B 446 E 459 C 472 A
408 C 421 E 434 D 447 E 460 B 473 C
409 B 422 D 435 A 448 B 461 A 474 B
410 C 423 A 436 E 449 C 462 D 475 B
411 E 424 A 437 C 450 D 463 A 476 A
412 E 425 D 438 F 451 B 464 D 477 B
413 C 426 E 439 B 452 C 465 B 478 A
414 A 427 A 440 A 453 A 466 A 479 A
415 C 428 C 441 E 454 C 467 A 480 B
416 B 429 C 442 A 455 A 468 E
417 C 430 B 443 C 456 B 469 A
418 A 431 D 444 D 457 A 470 A

a b c d e a b c d e
481 T F T F F 483 F F T T T
482 F F T T T 484 T T F F T

485 C 493 D 501 D 509 G 517 H 525 D


486 D 494 A 502 C 510 F 518 B 526 B
487 C 495 C 503 D 511 D 519 E 527 A
488 B 496 A 504 B 512 C 520 D 528 B
489 A 497 B 505 B 513 B 521 C 529 B
490 B 498 C 506 A 514 C 522 A
491 E 499 E 507 A 515 D 523 E
492 B 500 B 508 C 516 B 524 B

a b c d e a b c d e
530 F F F F F 532 F F T T T
531 T T T F F 533 F F T F T
534 B 543 C 552 A 561 D 570 B 579 A
535 E 544 E 553 B 562 D 571 D 580 B
536 D 545 B 554 D 563 C 572 D 581 B
537 C 546 A 555 B 564 B 573 C 582 B
538 E 547 D 556 D 565 E 574 E 583 A
539 A 548 E 557 B 566 B 575 D 584 D
540 E 549 E 558 B 567 D 576 C 585 B
541 C 550 A 559 C 568 B 577 D 586 B
542 C 551 C 560 D 569 D 578 C 587 A

a b c d e a b c d e
588 T T F F F 590 T T F F F
589 F F T T F 591 F T F F T

592 A 606 B 620 C 634 E 648 F 662 A


593 B 607 A 621 C 635 A 649 E 663 E
594 C 608 C 622 B 636 A 650 F 664 A
595 D 609 E 623 D 637 B 651 B 665 D
596 B 610 C 624 C 638 D 652 B 666 B
597 A 611 B 625 C 639 E 653 A 667 D
598 D 612 B 626 A 640 D 654 A 668 D
599 C 613 C 627 A 641 D 655 B 669 D
600 B 614 B 628 D 642 A 656 B 670 E
601 C 615 D 629 C 643 C 657 D 671 E
602 E 616 B 630 D 644 C 658 A 672 D
603 D 617 C 631 C 645 B 659 A 673 D
604 D 618 A 632 D 646 A 660 B 674 D
605 D 619 C 633 B 647 C 661 A

a b c d e a b c d e
675 F T F F T 683 T F F T T
676 F T T F T 684 F T F T T
677 F T T F T 685 T F F T T
678 F T T T F 686 T F T F T
679 T T T T F 687 T F F T F
680 F F F T T 688 T F T T T
681 T F F F T 689 T T T T F
682 T F F F T

690 B 699 C 708 B 717 A 726 B 735 A


691 B 700 C 709 A 718 B 727 A 736 E
692 A 701 B 710 C 719 D 728 B 737 A
693 E 702 A 711 D 720 D 729 A 738 E
694 C 703 B 712 C 721 C 730 A 739 A
695 D 704 D 713 B 722 E 731 E 740 A
696 A 705 B 714 D 723 D 732 A
697 B 706 D 715 C 724 C 733 A
698 E 707 D 716 D 725 B 734 A
a b c d e a b c d e
741 T F F T T 746 F T T F T
742 F F T T F 747 T T T T T
743 T F T T F 748 T F T T T
744 T F T F F 749 T T F T F
745 F 7 F F T

750 C 751 C 752 B 753 B 754 A 755 A

a b c d e a b c d e
756 T T F F T 758 F T T T F
757 T T F F T

759 D 763 D 767 D 771 C 775 A 779 A


760 D 764 A 768 E 772 D 776 D 780 E
761 A 765 E 769 C 773 A 777 D 781 B
762 E 766 C 770 A 774 C 778 E

a b c d e a b c d e
782 F T F F T 784 F F T T F
783 T' T F T T

You might also like